1st series [1] [2] [3] [4] [5] [6] [7] [8] [9] [10] [11] [12] [13] [14] [15] [16] [17] [18] [19] [20] [21] [22] [23] [24]  2nd series [1] [2] [3] [4] [5] [6] [7] [8] [9] [10] [11] [12] [13] [14] [15] [16] [17] [18] [19] [20] [21] [22] [23] [24] [25] [26] [27] [28] [29] [30] [31] [32] [33] [34] [35] [36] [37] [38] [39] [40] [41] [42] [43] [44] [45] [46] [47] [48] [49]

  View the latest questions and answers at askaphilosopher.org

Ask a Philosopher: Questions and Answers 39 (2nd series)

When referring to an answer on this page, please quote the page number followed by the answer number. The first answer on this page is 39/1.

(1) Melinda asked:

Are shadows real?

---

This is a nice question.

A shadow is not just a shape, because the same shape can be multiply instantiated. A shadow is a particular shape cast on a particular occasion. It occupies a spatio-temporal region, and arguably has continuity through time.

Shadows have causal properties. You can detect the movement of a shadow, or use a shadow to trigger a physical device.

It could be argued that these attributes suffice for being 'real'.

However, although a shadow possesses some notable properties of spatio-temporal particulars, it lacks the property of being made 'of' something. The shadow of my hands under my desk lamp, unlike the lamp, the computer keyboard, my mug of coffee has no substance of its own.

Some 'shadows' are capable of being captured and reproduced. We call the results 'photographs'. However, the undoubted reality of photographs as physical items does not in any way enhance the reality of the shadows that created them.

There is a problem which arises, however, when we consider the criteria for the identity of shadows. Imagine two runners passing one another in opposite directions. Each casts a shadow which moves as they move. When the shadows overlap are there two shadows are only one? I am inclined to say that there can be two shadows in the same place at the same time provided that they are cast by two light sources. The problem is that this contradicts what we would say about the criteria of identity for other spatio-temporal particulars.

Geoffrey Klempner

back

(2) Natalie asked:

I'm 15. I'm sorry if this does not seem a particularly 'philosophical' question, but I feel it is one that can only be answered by someone such as your good self.

As a 'teenager' who has begun to realise that mainstream culture and its values are not for her, instead wanting to improve her knowledge and understanding of the world, it has dawned on me (looking at this site) that I don't know where to begin when it comes to reading philosophy.

A lot of the ideas explored I do not understand, but don't know what to read first I started with Plato's Republic but of course, I did not understand many of the key ideas, and then went to the other extreme of a book which tried to incorporate philosophy into the Simpsons television series a painful attempt to say the least.

So, is there a starting place for an understanding of philosophy which you could recommend, and also, (just out of curiosity), how old were you when your interest began in the subject?

---

I got seriously interested in philosophy around the age of 21. I picked up lots of books, including Plato's Republic. However, my basic rule was, 'Stop, if you get bored.'

If you don't understand, that's not a reason to stop, because struggling to understand is part of philosophy. But if the text just doesn't grip you then try something else.

Imagine you are a detective on the track of the subject called 'philosophy'. When you get bored, the lead goes cold.

I'm not going to tell you which book you should start with because there is no such book. It really doesn't matter. Start anywhere. Start at the last page and read backwards, if that helps.

As you gradually get acclimatized to reading philosophy, you will gain the confidence (and the interest) to read a book all the way through from beginning to end. But don't do it just because you think you ought to.

Just out of interest: my middle daughter, 15, is currently reading Nietzsche's 'Thus Spake Zarathustra' and finding it gripping. It's not the book I would have chosen for her, but the important thing is that it is her choice.

Geoffrey Klempner

back

(3) Joe asked:

Are sound arguments true arguments?

Is it impossible to produce a valid argument for a false conclusion?

---

Probably yes for the first question and yes for the second but with reservations since what you mean isn't completely clear.

In general we don't talk about true arguments but only about logically valid arguments. Parts of these arguments I.e. Premises and conclusions can be described as true or false.

Here is a precise definition of a logically valid argument: A logically valid argument is one where if the premises are true then the conclusion cannot be false. Here is a simple example of that:

Premise 1 If p then q
Premise 2 p
Conclusion q

If we accept that premise 1 and premise 2 are true then there is no way that we can conceive of the conclusion being false.

This also answers your other question because that three line argument above will still be a logically valid argument even if the conclusion is false.

Here of course I am taking it for granted that your idea of a sound argument is the same as the idea of a logically valid argument. If it isn't then you must explain what you mean by a sound argument. People often confuse the idea of a logically valid argument with the idea of a sensible and relevant argument. Here for example is a logically valid argument that is neither sensible nor relevant.

If frozen peas always roll from left to right then birds have three wings and frozen peas do always roll from left to right therefore birds do have three wings.

It tells us nothing about frozen peas or birds but it's still a logically valid argument.

Shaun Williamson

back

(4) Emily asked:

Plato's Theaetetus:

What is the main question for which an answer is sought in this dialogue?

At 151de, Theaetetus offers a preliminary definition of the concept in question, but Socrates claims that this definition resembles that of a famous (infamous?) Sophist. Who is this sophist, and what is his theory about the concept in question?

---

I will answer these two questions to get you started in the right direction but I don't think it would be right for me to take your mid-term for you.

Plato's 'Theaetetus' is concerned with the nature of Knowledge. Socrates makes a couple proposals but ultimately seems dissatisfied with each answer and the dialogue ends before a conclusion is reached. The famous sophist he is referring to is Protagoras, who Socrates debates in an earlier dialogue.

Eric Zwickler

back

(5) Iain asked:

Has the private language argument advanced by Wittgenstein as a refutation of empiricist theories of mind + meaning itself been refuted?

---

These questions will always make me unpopular because I have to start by saying that there is no private language argument in Wittgenstein's work. The private language argument and the refutations of it are the inventions of philosophers who did not and do not understand 'Philosophical Investigations'.

Wittgenstein discusses the idea of a private language as advanced by philosophical sceptics (not empiricists) and he compares private languages with what we ordinarily think of as languages I.e. French, German etc.

However when you ask 'Has philosophical argument X been refuted' then this suggests that you think that philosophy is like science i.E it contains discredited theories. Philosophy isn't like science, there are no advances in philosophy, no accepted theories, no refuted theories. Philosophers agree about nothing. There are no accepted philosophical truths, everything is disputed. That is the peculiar nature of philosophy.

This is something that Wittgenstein understood and it is one of the reasons why his later work does not contain arguments. For him philosophy can't be advanced by arguing or by advancing new theories.

Shaun Williamson

back

(6) Gary asked:

Why do we have children?

---

Gary if you think about it then it is obvious that the inbuilt instinct to have children is as powerful as the instinct to eat or have sex. Evolution wouldn't work if it allowed a species to not want to have children.

If you don't want to have children yourself then it can be difficult to understand the force of this instinct. When I was younger I knew lots of women who told me that they didn't want to have children and would never want to have children. Then one by one they changed their minds. That is how humans work. We have instincts and we have to cope with them. Not everybody has to have children but the majority of people will have children and will want to have children.

Shaun Williamson

back

(7) 007 asked:

Does god exist?

---

This question isn't enough on its own since there are so many different ideas of what God is e.g. Aristotle's first cause, the loving God of Christianity, the extremist Christian or Islamic or Jewish God as someone who smites our enemies and makes us victorious over everyone who doesn't believe the same as us. There is the God who hates homosexuals, the Jehovah's witnesses god who will only save Jehovah's witnesses at the final judgement.
So which sort of God are you asking about?

Shaun Williamson

back

(8) Richard asked:

If I were going to experience/ live all possible lives through time, what would determine the order that I experience/ live them? Thanks

---

I have no idea how you expect anyone to answer this question. It doesn't describe a scenario with which we are all familiar. However logically you can only experience one life, your own. Any experiences you have are your experiences and therefore part of your life.

You may have experiences which are similar to those that other people have had but you cannot live more than one life, your own.

Shaun Williamson

back

(9) Richard asked:

Is harmony possible between all individuals (including lesser beings like crocodiles, and slugs), or do we have no choice but to want conflicting things?

---

It all depends on what you mean by harmony. You can live in harmony with a crocodile but if there is no other food available either you will eat the crocodile or it will eat you. The human may regret having to kill and eat the crocodile especially if it is an endangered species. However the crocodile doesn't suffer from such regret. To it a human is always just a potential meal.

Shaun Williamson

back

(10) Col asked:

Food chain question.

Is there a 'sum total' of life and as the human population grows does the rest of the sum total diminish.

We only eat living things or recently dead things or preserved things.

The way I see it there are rules about everything from gravity to DNA could the sum total of life be another 'rule'.

---

A famous chemistry experiment (the synthesis of Urea — look it up in Google) proved that 'organic' compounds can be synthesised from 'inorganic' compounds. In principle, human beings could live off 100 per cent synthesised 'food'. Whether we would want to is another matter.

What is true is that there is only a finite amount of matter on the surface of the earth. So at some point, there would only be human beings and nothing else, in which case we would have to eat one another.

Geoffrey Klempner

back

(11) Alexander asked:

Very sorry to trouble you via these means, but asked a question on your site yesterday and can't see to find it now. Thought id try a more direct approach as I urgently need an answer!

The question is:

By intervening in the world and the goings-on of humanity, is he interfering with free will? Obviously he isn't using his powers to directly change Mankind's abilities, but he is presenting or taking away options that will directly affect what Man decides to do.

The example I used to explain this was:

Allied troops during the Ardennes Offensive see a German tank approaching. They decide, as per free will, that they will stand and fight. God then intervenes and adds another 9 German tanks to the assault, inducing fear in the Americans and causing them to flee instead.

I argued that whilst God hasn't 'reprogrammed' and directly fiddled with their freewill, he has used outside stimuli to change their choices and reactions, thus meddling in Man's freewill. Couldn't get my friend to understand this, and him being a Philosophy graduate only served to make resistance that bit more stubborn!

Anything you can do to help will be very helpful!

---

In writing this email to you — when I could have chosen not to write — I am doing something which has an effect on the decisions that you make. For example, you might tell your friend that you received an email. In other words, my action is 'causing' you to do something that you would not otherwise have done.

Your example of God giving the Germans 9 more tanks is no different, in principle. It didn't have to be God. Anyone who had 9 tanks to spare and offered them to the Germans would have produced exactly the same effect on the Allied side.

However, the point in both cases, is that the ability to choose is not being restricted, only the circumstances in which the choice is made, or the number of choices available. Henry Ford once remarked that his customers 'can have any colour of car they like as long as it's black'. His marketing decision to only make black Model T's left car buyers with a restricted choice. But they were still free not to buy a Model T.

Geoffrey Klempner

back

(12) Patrick asked:

Not being a Christian, I've always been surprised by the obsession Christians have for God. Why is the knowledge of whether God exist or not so important? Why is praising God a pointless act besides the fact that it allows a social occasion and a chance to perform and boost one's self-esteem when others praise you for your talents so important? Do Christians do this because they want to have something to fill their time and boost their self-esteem or do they really mean to praise God because that's an significant and beneficial activity? In addition, I wonder why Christians are so happy about having an eternal life and so fearing about burning in hell. I personally would not want an eternal life because I would run out of things to do eventually and get bored. In addition, I don't think it make sense to have to burn in hell if you're not a Christian when in life you're not a bad person. And if I do burn in hell for not being a Christian, what's so painful about that? I'll adapt to the pain eventually through sensory adaptation, and by logic it doesn't make sense to have to burn eternally without dying. So why does it matter so much who's going to heaven and who's going to hell?

---

To start with, if you do go to hell (supposing there is such a place) then you won't adjust to the pain through sensory adaptation as you seem to think. The real pain of hell is being forever denied the opportunity to be in the presence of God and being aware just what it is that you have lost through this. Many of the people who ask questions about religion here, have a very basic idea of theology. However many Christians have a much more sophisticated idea of heaven and hell than you seem to imagine.

Having said all that I am an agnostic so I don't know if there is a God or a heaven or a hell nor do I care about such things.

It should be obvious that humans have a religious instinct and that this instinct is very strong. It has led people to make the most extraordinary sacrifices. It has also led them in to slaughter their fellow human beings who happen to have different religious beliefs.

Who is going to heaven and who is going to hell is very important to people who see their religion in terms of winning and losing. They thing they are going to heaven and God will smite all their enemies and send them to hell. Maybe God won't see things in the same way as they do.

Shaun Williamson

back

(13) Angel asked:

Is there a god?

---

I don't know.

Shaun Williamson

back

(14) Sophie asked:

Why do people live in areas where there are earthquakes

---

Earthquakes may not be very frequent in many of these areas I.e. They only happen every hundred years or so (outside living memory). The soil in volcanic areas can be very fertile. Its all a question of economic advantage. If people think the gamble is worth taking they will do it. Compare the question 'Why do people live on flood plains (New Orleans in the US etc.)'.

Shaun Williamson

back

(15) Hamed asked:

In The Nature of Morality Gilbert Harman argues that the fundamental problem of ethical judgments is their immunity from observation. We cannot see or observe moral wrongness or rightness.In your view, can the problem Harman identifies be solved?

(A) what is the problem Harman has?

(B) would a an utilitarian agree of disagree with Harman?

(C) your reasoned opinion of the significance of the problem Harman identifies.

---

It is a bizarre philosophical superstition to expect that ethical judgements should be anything like a observable truth. It reminds me of a quote from Wittgenstein, 'We are like primitive savages who take the expressions of civilised men and put a strange construction on them'.

So we take statements like 'This is red' and 'This is black' and compare them to statements like 'This is good' and 'This is bad.' And we try to understand one in terms of the other. However this is something that only philosophers do.

In real life people never confuse ethical statements with statements about sensory observation. We don't get confused about them, nor do we agonise about what they mean. Their meaning is obvious to everybody who hears them. SO what is it that philosophers fail to see?

Shaun Williamson

back

(16) Bruce asked:

Steven Nadler, in Spinoza's Ethics: An Introduction (Cambridge Introductions to Key Philosophical Texts), makes the following statement (p.85): 'Ordinarily, causal necessity is distinguished from logical or absolute necessity...Logical necessity would seem to be something that holds primarily between propositions, not things.' I have seen this de dictu/de re distinction made, as well, in objection to Hartshorne's revised ontological argument. I wonder about the justification for the distinction (other than its usefulness as a rejoinder to metaphysicians). Isn't it the point of reasoning about the world that logical relations may serve as a model for real relations? Leaving aside (if that is permissible) the question of alternative logics, is it not the case that the reason we find rules of logic to be valid is that reasoning according to these rules has consistently led to results that are verifiable in the world? Or is it now considered to be a well established principle that all logics are only systems and all truths valid only within a particular system? Surely, though, that's a theory and, if a prevalent position today, may not be so tomorrow.

I suppose I could rest content with this seeming division of philosophy from reality if it were made plain that what was being asserted by the denial of the possibility of inference to a world outside of propositions was only that there could not be absolute certainty regarding the conclusion (though the probability be ever so high)that is, that other rules of logic might one day prove to have been more appropriate or that some relevant factors were not considered.

To return briefly to Nadler, he gives, as an example of the distinction, the following: 'While we may regard it as unusual or contrary to some law of nature...For water to freeze at a temperature higher than 32 degrees Fahrenheit...We do not think that it is logically inconceivable.' That would seem to imply that the distinction is valid at a certain level of abstraction (I.e., When many facts pertinent to the physical situation, such as the nature of chemical bonds, are left out of the calculation). In the discussion of necessity in metaphysics, is one not usually meaning an unconditioned necessity, one presumably altogether without other conditioning factors to be considered (or to be abstracted from)? Does not the de re/de dictu distinction presume that only contingent relations are real?

---

I agree with you. It is true that logical necessities hold between propositions and causal necessities hold between things; but this is only a convenient distinction in the early analysis of this subject. More important is to ask what necessity is: if it is different in logic and causation then the distinction is important, and if not then not. There are two usual definitions of necessity. One is that necessity is universality: if A is always followed by B then A necessitates B, regardless of whether A and B are things (events) or propositions. The other is that necessity is singular possibility; as opposed to contingency, which is plural possibility; and impossibility, which is zero possibility. Then if, given that A exists, the existence of B is the only possibility, then the existence of A causes the existence of B; and if, given the truth of A, the truth is the only possibility, then the truth of A logically necessitates the truth of B.

(There is a moral parallel to this: what is morally allowed, required, or forbidden, is respectively of plural, singular, or zero moral possibility.)

Notice that if, given A, B is a singular possibility, then A will be universally followed by B: singular possibility necessitates universality, or universality is a singular possibility, given singular possibility. If you were to take course in statistics one of the first things you would learn is that correlation — even perfect correlation — is not causation. (e.g. Day causes night and night causes day.) Perfect correlation is universality, which is not singular possibility. Rigorous empiricists require that everything real is perceptible, so that for them necessity is universality because singular possibility cannot be perceived. So causation becomes perfect correlation and logical necessity becomes the universality of truth in a logical tautology. Notice that, ironically, in modern logic a tautology is a singular possibility of truth (namely, truth) while a contingency is a plural possibility of truth (namely, either true or false) and a contradiction is a zero possibility of truth (neither true nor false). The proper way to understand necessity is as a relation: a relation of singular possibility between antecedent and conclusion or between cause and effect.

Helier Robinson

back

(17) Rogelio asked:

If one has Psychological problems (I.e. Depression) goes to either the Psychologist or a clinic that has this type of assistance. But what about Philosophical problems (existence, sense of life, facing death etc...), Are there any Philosophical Consultant (Advisor) to follow a 'therapy'.

---

You have answered your own question just by asking it on this site: this site is a philosophy clinic.

Helier Robinson

back

(18) Lorie asked:

Assuming there is infinity, what form does it come in? It can't be a straight amount of years all straight things have a beginning. Does this mean infinite is just one big circle?

---

It could be a circle, since an infinite straight line may be thought of as a circle of infinite radius. Then its beginning and its end would be one and the same. My own view is that there is no infinity: it is just a word we use to cover our ignorance. When we don't know how big something is we call it infinite, just as when we don't know the cause of something we call it chance.

Helier Robinson

back

(19) Rodger asked:

Suppose I agree with presidential candidate Dennis Kucinich's substantive positions. Further suppose I am extremely bothered by the thought of him as president because I believe he might be mentally unbalanced, as evidenced by his repeated serious statement that he will become the next president. Does it make sense not to vote for him for that reason? Note that if I (and others in my position) vote for him I increase the likelihood of his 'mentally unbalanced' belief being true.

---

On the assumption that given the evidence from polls, Kucinich is very unlikely to become the next president, then his sincere belief that he will become the next president is evidence of irrationality.

The fact that Kucinich's chances of election increase if you vote for him is irrelevant, since we judge a person's rationality in relation to evidence available to that person.

For example, if I believe that I am going to be hit by a meteorite, this belief is irrational even if the billion-to-one chance occurs and I actually am hit by a meteorite.

Geoffrey Klempner

back

(20) Joel asked:

Hi, I am a philosophy major in year one trying to sort out my ideas clearly. I would like to clarify my idea about the idea of toleration in Mill. My professor has asked us to write an essay on the Mill's conceptualization of toleration as a virtue in On Liberty. I have a framework for the paper so far by proposing that Mill's Harm principle and theory of utilitarianism is underpinned by the concept of toleration as a virtue.

As such, for Mill's concept of the liberal state to exist, toleration is a virtue that is necessary. It is not enough to see toleration as serving one's interest, it is necessary that we consider all opinions in order for the individual and consequently the state to grow and be free. As such toleration is a virtue linked with the classical virtues of humility (Mill does seem to suggest this in his discussion) and intolerance with pride and conceit.

I would like to know how strong this argument is and whether or not I should include the idea of how toleration is the benchmark for the liberty of the individual and if the individual hopes to be a free rational being toleration is thus a virtue that allows him to carry his free will without infringing or imposing himself on others. Thank you for your help!

---

Your idea of linking toleration with humility sounds too Christian for Mill. For Mill, 'humility' is a purely intellectual virtue of being prepared to be proved wrong. That is very different from what Christ (allegedly) meant by 'humility'.

Mill thought that all religions — including Christianity — ought to be prepared to argue their claims out in the court of reason. That is what 'freedom of opinion' is all about. The problem is that this fatally handicaps any moral view which depends on received teaching or religious faith. This, arguably, is the most serious weakness in Mill's case for so-called 'tolerance'.

For a provocative take on this, see Maurice Cowling 'Mill and Liberalism', 2nd Edn. Cambridge 1990.

See also my paper, 'Ethical dialogue and the limits of tolerance' http://klempner.freeshell.org/articles/tolerance.html

Geoffrey Klempner

back

(21) Cherise asked:

What is the meaning of 'virtue is its own reward'.

---

Virtue is its own reward if it is true that virtuous acts cause spiritual development in the virtuous person, and if it is true that spiritual development is desirable.

Helier Robinson

back

(22) Lester asked:

What is equivocation?

---

Equivocation is using a word with two or more meanings as if it had only one meaning. For example, the word 'same' can mean similar or identical. Similarity is a relation which holds between things that resemble each other in one or more ways; while identity is oneness, as in everything being identical with itself, or one and the same with what it is. (Often 'identity' is used to mean exact similarity, but this is another equivocation.) One might then argue that I yesterday am very similar to me today, therefore I yesterday is the same as I today, therefore I yesterday is identical with I today. The argument commits the fallacy of equivocation because of equivocating between the two meaning of 'same'. Notice that identity entails exact self-similarity but exact similarity does not entail identity.

Helier Robinson

back

(23) Kelly asked:

What is an adequate solution and what is the adequate solution to the problem of evil?

---

Logically there are several solutions: God is not all-loving and does not want to abolish evil; God is not all-powerful, so is unable to abolish it; God is not all-knowing, so does not know that evil exists; and there are two gods, one all-powerful but not all-loving and one all-loving but not all-powerful. The usual explanation that evil exists for the sake of a greater good works only if that greater good can be spelled out and shown to be impossible without the existence of evil; and the usual claim that free will is such a good does not meet these criteria — particularly since an all-powerful and all-loving God could have created us so as to have free will yet never commit an evil act — as angels are reputed to be. Another explanation is that evil is sent by God to try us, to separate the sheep from the goats; but why would an all-knowing God need to do this?

Helier Robinson

back

(24) Charlotte asked:

I'd like to explore the idea that the ultimate reality behind the universe is mathematical. Would you please send me some thoughts on this.

---

Your question suggests that you are going to be a very good philosopher. But better to say that the ultimate reality is described by mathematics, rather than is mathematics. And, yes, it probably is — simply because all the most successful sciences, such as astronomy, physics, and chemistry, are mathematical. You then have to ask what it is that mathematics describes. Although the basics of mathematics have been sets, for the past century or so, these are not what math describes. For example, Newton's laws do not describe sets; nor does quantum mechanics or the theories of relativity.

In fact the answer to this question is relations: relations of space and time, causal necessity, similarity and difference, pattern and order, etc. Relations have always given philosophers a lot of trouble; for example, can we perceive them, when they do not have any colours or other concrete qualities? And if they are abstract entities, why is the world we perceive concrete? There are possible answers to all this in my 'Relation Philosophy of mathematics, Science, and Mind' which can be downloaded for free from http://www.sharebooks.ca, but if you want to learn to philosophise for yourself you would do better to get a job that gives you plenty of time to think: a lighthouse keeper or a forest fire watcher. Then you can practice thinking on your own instead of cluttering up your mind with other people's ideas, and only then read other people's writings.

Helier Robinson

back

(25) Slim asked:

What is a material conditional in logic and what is important to note about it as it relates to language?

---

A conditional is an argument of the form 'If... Then...' For example, 'If Pat is pregnant the Pat is a woman' the first proposition is called the antecedent of the conditional and the second is called the consequent. In modern truth-functional logic, compound propositions such as conditionals are defined by truth tables: that is, a conditional is true unless its antecedent is true and its consequent false. This definition leads to the so-called paradoxes of material implication: a true proposition is implied by any proposition and a false proposition implies any proposition. Since we do not ever argue logically in this way, the truth-functional conditional is called a material conditional (or implication), to distinguish it from genuine conditionals.

Helier Robinson

back

(26) Lee asked:

Can we doubt our existence without existing while we doubt? Does this mean that we exist? This question confuses me quite a bit! I mean you have to exist to have the ability to doubt anything, because if you doubt something, that means that you in fact have an opinion and how is it possible to have an opinion without some sort of knowledge? To have knowledge you must exist right?

---

Yes, you are quite right. You have to exist in order to doubt. This was Descartes' method of finding indubitable truth: I think (I.e. Am conscious) therefore I am (I.e. Exist). The trouble with this starting point is that you cannot get any further from it: if you want certainty then you are stuck in solipsism, which is the belief that I alone exist — nothing outside my present consciousness exists.

Helier Robinson

back

(27) Marianne asked:

Many have heard of the strange case in Germany when a man killed and ate a voluntary victim (Armin Miewes). If it is consensual, why is it wrong in the context of law? If not, are there grounds for legalizing consensual cannibalism?

---

To kill someone even if they ask you to kill them is murder. Of course in some countries exceptions are made for euthanasia. In the case that you refer to I believe that it was the crime of murder that was the chief offence. I'm not sure that most countries have laws against cannibalism but they certainly have laws against murder, soliciting murder, assisting suicide etc. And I don't see any reason why this should be changed.

I can't see any reason for legalising consensual cannibalism if in fact it is illegal (when murder isn't involved). If someone wants to eat meat let them go to a supermarket and buy pork. I am told it has a texture and taste that is close to the flesh of humans. There are good medical reasons for humans not eating human flesh. If you don't know what these are then you should do some research on things such as BSE and cannibalism.

Shaun Williamson

back

(28) Robert asked:

If one person out of the 7 billion people on earth, could legitimately know, see and predict the future. Would that eliminate free will for all of us?

---

I can see the point to this question. If one person could know and predict the future with perfect accuracy — and leaving aside what this would mean for the person concerned in terms of his/ her ability to formulate and carry out plans of action — it would mean that there is something determinate 'to be known' about the future, a fact about how things will be.

In that case, whatever anyone of the remaining 7 billion decides to do or not do, it is already a fact what they will do before they make their decision.

There are theories of human action which would say that there would still be a point in distinguishing actions for which we are responsible for and actions which we are not responsible for, that is to say, in a sense there is a kind of 'freedom' which is compatible with the future being fully determinate. However, arguably, it is not the kind of 'freedom' that we want. If you were convinced that someone really did know everything you were going to do before you decided to do it, that would be a devastating discovery.

However, I would not accept the premiss of your question. As a matter of fact, I do not accept that it is possible — that is to say, consistent with the laws of physics — for any being to 'legitimately know, see and predict the future'. All knowledge is ultimately based on measurement, and all measurement has a margin of error. The further ahead one tries to predict, the more errors are multiplied. Given this limitation, there is no way — short of intervention by a divine being which I am not considering — that such knowledge could be acquired.

It is still a problem, though I would argue a lesser problem, if one believes in causal determinism. In that case, the future is determined even if no person can ever know the future.

Geoffrey Klempner

back

(29) Petros asked:

Have philosophers failed the social/ moral test?

I feel that philosophical ideas have been alienated from the practicalities of society. Philosophers should be at the cutting edge of social activities and highly visible and accessible. They should be working to challenge the evils of the world such as racism, war, greed and other dehumanising aspects of social organisation and conflict. What is the point of all this 'love for wisdom' when it is restricted to academia and the odd popular book that leaks into the public domain? Philosophers should be running for political office, on boards in financial institutions and influencing the way science is conducted. Philosophers should be our modern day cerebral and spiritual guides. If they hide in the safe confines of complex terminological discourse and cult temples of passage, the discipline of philosophy merely renders itself as 'foolosophy' the wisdom of foolishness. For what point is there to have all this grounding in wisdom when it is locked up in some self serving bubble of elitism and exclusivity?

---

I agree.

Geoffrey Klempner

back

(30) Brian asked:

There is a very popular movie being marketed at present called 'The Secret'. It's basic premise is what's been called The Law Of Attraction, ie if we stay focussed on what we want then it magically appears (manifests). What do you think of that idea? Is there anything about this so called Law in any of the wisdom literature down through the ages? One of my issues with this theory is it seems to perpetuate the idea that external stuff is needed for happiness.

---

Brian, take a look at the Wikipedia entry for 'Law of Attraction', and see the history and various criticisms of it. In my opinion, how you think about the law determines whether there is anything to it. If you interpret the law as 'wish hard and long enough, and your wish will come true' then I would say it is bunkum. Such an interpretation demands a causative connection between your wish and the outcome. There is absolutely no evidence to support such a causative connection. On the other hand, if you interpret the law as 'focus your intellect and energies on what you want, and you can make it come to pass', then I think the law is a simple statement of 'you get what you work for'. This interpretation has the pleasant benefits of being obviously true, and not violating any science.

Stuart Burns

back

(31) Jonathan asked:

I am wondering is belief involved in pure science?

I am a scientist and a philosopher who frequently meet people who claim this is the case. I have tons of arguments against their idea. But never manages to convince them the bastards.

---

Jonathan, what you have to do is clarify just what is being referred to by the word 'belief'. Two separate senses are often confounded by those who argue that science is just as founded on 'belief' as is religion. The two different senses of 'belief' can be described as (a) confidence in the truth or existence of something not immediately susceptible to rigorous proof, but supported by the available evidence; and (b) confidence in the truth or existence of something in the absence of any supporting evidence. Science demands belief in its processes and results in the first sense. You, as an individual scientist, do not have available to you anything like rigorous proof of the truth of the various statements of science. But you do have adequate evidence to justify your beliefs beyond the point of unsubstantiated opinion. By contrast, religious beliefs are held as true in the absence of any supporting evidence at all. The difference then is the degree of supporting evidence.

Stuart Burns

back

(32) Emily asked:

Plato's Theaetetus:

What is the main question for which an answer is sought in this dialogue?

---

The Theaetetus of Plato is considered to be his most important work on Epistemology, since it deals with the definition of Knowledge itself. It was written after the Parmenides, where Plato criticized his Theory of Forms, therefore Plato proceeds here without the use of any Forms to the definition of Knowledge. After three attempts of definition the Dialogue ends in failure, since the knowledge has not been defined and the conclusion that really comes out is 'what is not knowledge', certainly without implying that Plato defines 'knowledge' through privation of the opposite term.

This Dialogue, on the one hand, by examining all the epistemological arguments of its time (Protagoras, Heraclitus, Empedocles and the Megarians) increases the reader's awareness of knowledge, since it explores all the concepts relative to knowledge (perception, belief, definitions, prepositions, misidentification, memory, relation between complexes and its elements etc.) And triggers off further philosophical debates on epistemology. On the other hand, in my opinion, here Plato proves the falsity of the epistemological notions of his time, as well as the impossibility of the definition of knowledge without the use of the Forms or of Dialectic. Therefore the definition of the Sophist and the Statesman is possible in those later Dialogues by the use of Dialectic.

The dialogue takes place mainly between Socrates and Theaetetus, a disciple of Theodorus the great geometrician, whose science is thus indicated to be the propaedeutic to philosophy. It is an enquiry into the nature of knowledge, which is interrupted by two digressions. The first is the digression about the midwives (149-151), where Socrates is supposed to have a mission to convict men of self-conceit; and has assigned to him by God the functions of a man-midwife, who delivers men of their thoughts, and under this character he is present throughout the dialogue. He is the true prophet who has an insight into the natures of men, and can divine their future. The second digression (172-177) is the famous antithesis between the sophist, statesman, lawyer on the one hand and philosopher on the other hand. The philosopher naturally desires to pour forth the thoughts, which are always present to him, and to discourse of the higher life. The idea of knowledge, although hard to be defined, is realized in the life of philosophy.

In the Theaetetus, although Socrates uses his maieutiki (midwifery) method, he fails to define the knowledge. The Dialogue begins with the question made by Socrates 'what is knowledge', and aims at giving an account of it, but after three proposed attempts of definition by Theaetetus, it ends in 'aporia'.The three proposed definitions are: 'knowledge (episteme) is sensible perception' (aesthesis), 'knowledge is true belief' (doxa alethes) and 'knowledge is true belief with an account' (doxa alethes meta logou). Plato through Socrates proves that the three definitions are not sufficient; therefore in the end knowledge has not been defined.

Scholars have argued about the aim of this Dialogue, as well as about the timing that was written. Unitarians (Aristotle, Proclus, Bishop Berkeley, Schleiermacher, Ast, Shorey, Dies, Ross, Cornford, and Cherniss) on the one hand, claim that Plato intends with it to show 'what is not knowledge', as well as the impossibility of definition without the use of the Forms or of Dialectic. Apart from that they argue that Plato leaves open the third definition of knowledge 'true belief with an account' for further exploration and development in later Dialogues. This implies that the account (logos) presupposes the knowledge of the five great kinds (Being, Sameness, Difference, Motion and Rest) and their ability of blending, as it comes out of the later Dialogues Sophist and Statesman. Revisionists (Lutoslawski, Ryle, Robinson, Runciman, Owen, McDowell, Bostock, Burnyeat etc.) On the other hand, claim that Plato, after having criticized his theory of Forms in the Parmenides, proceeds in all philosophical questions without the use of any Forms, and especially this Dialogue shows the change of Plato's method, for the five great kinds, mentioned in the Dialogues after the Theaetetus, are not clearly specified as Forms.

Nikolaos Bakalis

back

(33) Emily asked:

At 151de, Theaetetus offers a preliminary definition of the concept in question, but Socrates claims that this definition resembles that of a famous (infamous?) Sophist. Who is this sophist, and what is his theory about the concept in question?

In his preliminary objections to this sophists view, Socrates wonders why not just exchange his definition with animals such as pig or baboon. Why does Socrates make this claim, and what is he trying to imply by it?

How does the concept of Heraclitean fluxism, which Plato believes to be true of the material world, provide an argument against the sophists view of the concept in question?

In his first major argument, Socrates attempts to show that this sophists claim is self-refuting. Give at least one reason why it is.

---

The first attempt of definition by Theaetetus is the empirical one; so to say knowledge is perception, which might mean sensory awareness. The response of Socrates is to connect this definition with the relativity of the sensual perception that claimed the sophist Protagoras — man is the measure of all things — by the use of the example of the 'cold or not wind'. The wind appears to some people cold and to some not but the wind in itself is different from that relative perception. Therefore he states:

'Now is the wind, regarded not in relation to us but absolutely, cold or not; or are we to say, with Protagoras, that the wind is cold to him who is cold, and not to him who is not?' (Plato's Theaetetus 152b).

Apart from, that he tries to show that according to this account it follows that:

1. There is no independent quality in itself. Qualities have no independent existence in time and space (153d-e).

2. Qualities do not exist except in perceptions of them (153e-154a).

3. The dice paradox (154c): changes in a thing's qualities are not so much changes in that thing as in perceptions of that thing.

These three hypotheses lead inevitably to the Theory of Flux of Heraclitus, therefore Plato's strategy is now to connect this definition of Protagoras with the one of Heraclitus in order to prove the falsity of Protagoras' notion by the use of the Heraclitean arguments of flux, as we can see.

'I am about to speak of a high argument, in which all things are said to be relative; you cannot rightly call anything by any name, such as great or small, heavy or light, for the great will be small and the heavy light — there is no single thing or quality, but out of motion and change and admixture all things are becoming relatively to one another, which 'becoming' is by us incorrectly called being, but is really becoming, for nothing ever is, but all things are becoming.' (Plato's Theaetetus 152d-e).

This account however, could imply that either everything is in flux or only the objects of perception are in flux. Therefore Plato has to make clear all these puzzles by raising different objections against Theaetetus' definition. At first he starts to develop the Protagorean-Heraclitean account of perception by the use of the theory of flux. The ontology of the flux theory distinguishes kinds of 'process' (kinesis), I.e., Of flux, in two ways: as fast or slow, and as active or passive. Hence there are four such processes. On these the flux theory's account of perception rests. Therefore he states:

'Their first principle is, that all is motion, and upon this all the affections of which we were just now speaking, are supposed to depend: there is nothing but motion, which has two forms, one active and the other passive, both in endless number; and out of the union and friction of them there is generated a progeny endless in number, having two forms, sense and the object of sense, which are ever breaking forth and coming to the birth at the same moment.'

Following this account, the percipient (passive) and the object of perception (active) meet each other and create the senses of seeing, hearing, smelling, tasting and touching. And every time these two factors are associated, another sense is generated. This means that senses are no independent entities in themselves, but only relative in space and time, for every time different senses arise out of this association, both in different persons and in the same person. Socrates returns to this point later in 181b-183b, after the refutations of 'knowledge is perception'.

Plato's aim is now to show first which of the two accounts is right, so to say everything is in flux or only the objects of perception are in flux. I believe that Plato considers the latter premise as true. Therefore he examines through Socrates the Heraclitean principle of motion (kinesis), and distinguishes two kinds of motion, namely slower and quicker or else alteration and spatial motion. Moreover, he presents three contradictory premises of the flux theory to prove the relativity of the sense theory, 1. Nothing can become greater or less, either in number or magnitude, while remaining equal to itself, 2. Without addition or subtraction there is no increase or diminution of anything, but only equality, and 3. What was not before cannot be afterwards, without becoming and having become (155a-b).

In order to support the paradox of the three premises he gives the example of the dice, and of the comparison of Socrates with Theaetetus in different timing. To be more specific, the six dice are more by a half when compared with four, and fewer by a half than twelve, which means the dice six are both fewer and more by half at the same time (154c). On the other hand, Socrates who is of a certain height and taller than Theaetetus, may within a year, without gaining or losing in height, be not so tall (premise1) — not that he should have lost, but that Theaetetus would have increased. In such a case, Socrates is afterwards what he was not once (pr.3), And yet he has not become; for he could not have become without becoming, neither could he have become less without losing (pr.2) Somewhat of his height (155b-c).

In both cases the perception of the entities magnitude and height are proved relative and not independent in space and time. By the use of these Heraclitean paradoxes, Plato tries to prove the inconsistency of Protagoras' notion. Following the theory of flux, and the senses regarded as a result of association of the passive (percipient) with active (object of perception) agent, there must be no difference between thoughts and knowledge arising from perceptions in dreams, perceptions of healthy, sick or mad man.

1 Perceptions in dreams: If all perceptions are true, then it follows that the thoughts and judgements arising from those perceptions in both states (sleep and awake) must be also true, although they are contradictory.

2. Perceptions in madness: Plato distinguishes this state from the previous only in relation to the quantity of time, since in both states thoughts are contradictory, however there is more time present in mental disorder than in dreams.

3. Perceptions of healthy and sick men: By the use of the example of tasting the wine in relation to 'Socrates healthy' and 'Socrates sick', Plato proves according to Heraclitean theory of flux the relativity of the perception of taste. So to say, when Socrates is healthy the wine tastes sweet to him, while when he is sick it tastes bitter. Qualities then of sweetness and bitterness are no independent in themselves, but only in relation to the percipient.

After the interference of Theodorus, the first objection of Socrates to Protagoras' notion is that: if all perceptions are true, then it follows that there is no reason to think that animal perceptions such as pig or baboon are inferior to the human ones (161d). However, since the Protagorean view could mean that 'the opinion or the judgement formed on the basis of perception is true', he objects that this implies that 'no one is wiser than anyone', which means even Protagoras himself (second objection). The third objection, which follows, is that the human perceptions should not be inferior to the gods' as well, which is also absurd (161a-162d).

Then Socrates sketches the response of Protagoras to his objections, which is focused on Protagoras' therapeutic model of teaching as follows: There are no true and false beliefs, but only different judgements, and what the wiser man (Protagoras) does is to change by the use of arguments — just like the good doctor by the use of drugs — the state of the pupil's soul, in which 'bad things are and appear', into the one in which 'good things are and appear'. For while all beliefs are true, some are more beneficial (better) (Theaetetus 167 a-c).

The response of Socrates to this objection is the well-known 'peritrope' (table-turning) argument as follows: Protagoras admits that 'his opinion is true' and also that 'the contrary opinion' that the other people might have 'is also true'.

'And in conceding the truth of the opinion of those who think him wrong, he is really admitting the falsity of his own opinion' (Plato Theaetetus 171b).

In other words, 'all beliefs are true' is true, and 'not all beliefs are true' is also true. In this way, according to Plato, Protagoras contradicts himself by refuting his own argument, therefore he cannot claim that his beliefs are more beneficial than the others'.

Nikolaos Bakalis

back

(34) Karen asked:

Who/ what kinds of groups decide what is ethical?

---

We all decide. Everyone is morally equal. Morality is not like politics. Everyone has an equal say and an equal responsibility. There are no groups or individuals who have a privileged position when it comes to deciding what is ethical.

Shaun Williamson

back

(35) Edroger asked:

I am looking for answers of this question.

How does different way of thinking of different civilization, affect the way the society perceive space in art.

e.g. Deep space, one point perspective in western art, and shallow space or flat space, conceptual space in eastern art.

---

This is a complex question that you will only really understand if you study the history of art. The idea that art should represent a copy of three dimensional reality on a two dimensional canvas was an idea that has a relatively brief history (of about 500 years or so) and only applies to pictures, sculptures were always three dimensional. However learning to represent three dimensions on a flat surface had important consequences for technology. It made possible accurate engineering drawings for example.

There are complex reasons why other civilisations arrived at their idea of what art should be. It is not a simple contrast between western and eastern art. Look at Picasso and the cubists for example. Why do people in Egyptian art always have two left feet there is a good reason for this.

Shaun Williamson

back

(36) Jed asked:

How appropriate is the existential and essentialist affects or points of view on our life? I know that existentialism points out 'existence precedes essence' and in essentialism 'essence precedes existence'. I really don't get much about these views...It is very broad.

---

Existentialism highlights human free will by maintaining that existence precedes essence — we create who we are, we are not hindered by or determined by any pre-existing essence or human nature. This is found most famously in the writings of Jean-Paul Sartre. In a different guise, it is found in the writings of Michel Foucault but nevertheless, the point remains the denial of any pre-existing and determining human essence or nature.

Essentialists maintain there is a human essence or nature which determines how we act. Ludwig Feuerbach and Karl Marx maintain there is a human essence but it is alienated by present society. Reconciliation with the essence is the aim of good politics and identical with the process of human history.

Essentialists

Just as the seed has the potential to become the tree — the tree being the final stage in the process that began with the seed — so the human essence is the seed to the subsequent growth and development of the person. Essentialists deny they are determinists. Humanist Marxists will point out that only human activity can overcome alienation through revolutionary change. Human beings must at least make the choice to do this and this very act of choosing is incompatible with determinism thereby refuting it. Aristotle would maintain that understanding the essence or nature of a person allows the person to lead the good life by informing his actions with reference to his nature. Acting in accordance with one's essence, one rationally maintains a balance or mean in all things. One ought to aim for a life of contemplation [Theoria] as this is acting out the defining essence of human beings — ratio or Thought. Of course, not everyone will do this and they will fail to lead the good life. In both cases the existence of an Essence does not determine subsequent action, it merely provides a blueprint to follow if a person is to live eminently, virtuously and excellently. Essentialists would ask Existentialists that if you maintain there is no essence then what precisely makes choices? If there is no subject then what agent makes choices and, if there is no agent then indeterminism ensues — there are no choices only inchoate, unrelated, capricious actions.

Existentialists

Existentialists maintain that the existence of an essence prohibits freely choosing actions. If Essentialist's say there is an essence but it doesn't determine then it isn't really an essence by definition. Besides, if the essence can only be accessed by freely choosing then the existentialist case is made — choice is paramount in creating what or who one is. Can you have an essence by choice? No — no essence is opted for, only a choice made and a different option chosen. If essence determines persons' actions then agency is problematised. There will not be an agent that freely chooses courses of actions. If no agency then there is no responsibility for actions. A person would be no more than a wound up clockwork device dispensing actions and responses when 'unwinding'. I will not be talking to a person with his/her own views but talking to a clockwork device with clockwork responses. This follows out of the proposition that essence precedes existence. If a person declares 'I am my essence', who or what is declaring this? To be aware of something means to be conscious of it as an issue before me. If it is 'before me' then I am not it — I am not identical with it. If not identical with it then I'm apart from it. If apart from it, there is a discontinuous space between it and myself. This space — because it is nothing — is not hindered or determined by anything. As such it is a space for Freedom — Freedom to act. In Sartre's terminology, this is Nothingness. Nothingness dialectically interacts with Being thereby ensuring Freedom. Each is evanescent to the other as they interact.

Martin Jenkins

back

(37) Adam asked:

Are there limits to science and rationality?

I heard a physics teacher talk about the impossibility of modelling feedback mechanisms and this underpinning elements of chaos theory.

If I understood this then surely when inventing and testing drugs for example those that target a particular pathway, outside the controlled environment of a test tube which looks at one thing in detail; but when used in an animal where many pathways involve feedback mechanisms we couldn't predict the results. Would this explain why clinical tests with drugs can give such varied results.

Then doesn't this act as a natural endpoint for science?

---

What your examples show is that there are limits to human knowledge. Why should that be surprising? The fact that there are certain 'facts' which we can never know means that we will never become omniscient. It does not show that there is a 'limit to science' in the sense of an outer limit to what we can discover through applying the scientific method.

Think of it this way: there are two kinds of 'limits', those that define how much we can investigate at any given stage of inquiry — either because of limited resources or for more fundamental mathematical reasons such as the ones you cite in relation to chaos theory — and those which serve as an ultimate barrier to progress. What I am saying is that in the case of science, there is no evidence that such barrier has been reached, or will ever be reached.

Geoffrey Klempner

back

(38) Abben asked:

My friends and I played this game called Nomic, where players are given a set of rules to play by, and spend one turn each proposing and voting on changes to that set of rules.

The game ends either (1) when someone 'wins' as defined by the rules or (2) when the rules get so contradictory it is impossible to follow them.

Right now, (2) might have happened to us, but we aren't sure.

This is the rule in question:

'The player with the lowest score may switch seats with anyone else, one time, when they have finished their turn.'

This rule explicitly refers to a single player. But, in our game, two players ended up tied for the lowest score. Thus there is no single player with the lowest score, and we are unable to obey this rule because it is a rule that is attempting to apply to a nonexistent player (that individual player with the lowest score).

On one side of the debate, players are arguing that since this 'single player with the lowest score' is undefined, it is impossible to follow this rule. We are forced to obey this rule and yet we can't because we don't know who to apply it to.

There is an opposing side, arguing that since there is no single player for the rule to apply to, the rule simply does not apply at all. It waits in the background for 'the single player with the lowest score' to become defined, and then it applies because that player exists.

I think, then, that we have two possibilities:

(1) Is it inherent in the rule that the conditions wherein it applies must always exist, or (2) is the rule itself conditional, applying only when such conditions exist that it can apply?

Or, do we have a third choice, namely, that we can not decide upon one or the other because the rules aren't clear enough?

Or, maybe there is even a fourth option: Even if the rule is too vague we should be able to look at the other rules and the manner in which they are supposed to apply. Can we just look to the assumed agreements we have already made and find a hidden precedent somewhere which tells us how to proceed?

Assume for the sake of discussion that there are no other problems with the rule besides the one we have identified, and that none of the other existing rules tell us what to do in this situation.

I would not be surprised if the response is that we can't decide one way or another, it depends upon other rules or how we choose to define X or Y, but it would nonetheless be interesting to hear if there are any clear answers, or at the least, it would be nice to be told where we could look for an answer.

---

The question whether your practice or the other rules might set a precedent for interpreting the rule, 'The player who is so-and-so does such-and-such' seems to be a red herring.

On the assumption that the strict wording of rules is important, then we can rule out the interpretation, 'Any player who is so-and-so does such-and-such'. If the person who formulated the rule had meant, 'any' they should have said 'any'. 'Any' does not mean the same as 'the'.

The remaining choice is between:

(a) 'If at any time there is one and only one player who is so-and-so then he/ she does such-and-such.'

(B) 'There will always be one and only one player who is so-and-so, and this then entitles him/ her to do such-and-such.'

Well, imagine the proposed rule had been, 'The player with a score of 30 may switch seats with anyone else, one time, when they have finished their turn.' It would have been immediately obvious to anybody that this rule as failed to allow for the possibility of either none, or two or more people with a score of 30.

By parity of reasoning, we should say exactly the same thing about 'The player with the lowest score' even though the wording made the fault less obvious.

It follows that the correct interpretation is (b). The rule is impossible to apply because not properly defined.

Geoffrey Klempner

back

(39) Erica asked:

Assuming that Psychological Egoism is true, why might this imply the truth of Ethical Relativism?

---

In 'The Ego and Its Own' Max Stirner [1806 — 1856] maintained that the only important things were the Ego and its Ownness. It is not solipsism but a prioritising of the Ego against everything else.

Spooks

For Stirner, there is no God as First Cause and as creator of human beings. Following from this, there is no Soul. Following from this there is no secularised soul in the guise of a Universal Human Essence belonging to us all and from which, moral principles and prescriptions can be drawn whether humanist, liberal or socialist or capitalist. So authority drawn from such fictions or 'spooks' as Stirner calls them is not to be recognised by the Egoist. Although they evoke origin from a 'Higher Power' the 'Spooks' are external and not authorised by the Ego. Having grown from erroneous thinking and becoming 'Fixed Ideas' determining the way people think, the Egoist denies them. S/He recognises that 'Spooks' are the intellectual Creations of humanity which reified, have become the Masters of Humanity. Transforming this inversion thereby exorcising the Spooks the starting point of all human relations is finally recognised as the Ego.

I am my World

I or Ownness is the origin of all action. The Ego secures what s/he wants literally by any means necessary. Might and whatever else is at the Egos disposal facilitates getting what s/he wants. If another fails to get what s/he wants well tough, try again or give up. As the realm of 'Higher Truths' and 'Essences' has been negated, there are no Absolute Truths. Whatever the Ego has power to access, s/he has right to.

'I decide whether it is right thing in me; there is no right outside me. If it
is right for me, it is right.' [P.248 Ego]

This does not preclude Ego's co-operating with each other for specific goals; Stirner stipulates that Ego's can voluntarily combine in a Union of Ego's to achieve mutual ends, services, interests.

Relativism?

So prima facie Stirner would be inclined toward Ethical Relativism. Denying a single source of ethics from a Transcendent or Humanistic source, Stirner cites the source of valuation in the Ego. Its Power or the ability to get what it wants permits it to value and label what it wants how it wants: this is Ownness. This autobiographical definition of ethics sits easily with extreme ethical relativism.

However, if the Ego co-operates with other Ego's in a Union of Ego's, extreme Egoism gives way to Utilitarian based Egoism. Values and Ethics are understood and shared 'collectively'. This permits a logical development that following definite ethics and values can satisfy the greatest interests of the greatest number of Ego's. So we voluntarily agree it is better not to murder, better not to steal and so on. Paradoxically, Psychological Egoism can, in certain instances, achieve more by utilitarian combination that by isolation. The latter possibility demonstrates that Psychological Egoism does not of necessity, entail Ethical Relativism.

Martin Jenkins

back

(40) Erica asked:

Sartre takes atheism as a sufficient condition for man's existential situation. Why does he think this? Is Atheism a necessary condition for our situation?

---

Existentialism is the philosophy of freedom developed from a consistently atheist position. Why atheism? The traditional, orthodox Christian God is almighty, all-powerful — omnipotent. Applied to knowledge, the omnipotent God must necessarily be omniscient: God knows everything. Knowing what happened in the past, happens in the present and especially what will happen in the future, His foreknowledge can not be contradicted. If contradicted God would be wrong and this is impossible. So if God knew in 2000 that I would answer this question in 2007, I could not have acted otherwise. I have no freedom to choose. If Existentialism is the philosophy of freedom then it is irreconcilable with a belief in God — the Orthodox Christian God at the very least. God prevents the freedom that is essential for the human freedom propounded by Existentialism. Without the sufficient condition of atheism — which guarantee's freedom by denying the existence of God — humans cannot be free. For Sartre, Freedom defines the human condition.

Martin Jenkins

back

(41) Jeff asked:

There is a law that states the easiest explanation is the simplest. I think its called Achbar's razor?

---

It's called Ockham's razor after the English Medieval philosopher William of Ockham. His rule is that we should not complicate explanations unnecessarily I.e. The simplest explanation that fits the facts is probably the best.

Shaun Williamson

back

(42) John asked:

Why does man need god?

He doesn't. The many disbelievers in God prove that.

Stuart Burns

back

(43) Paul asked:

I have two paradoxes which I am trying to disprove. They were both given me by a maths teacher in the early 1980s.

1 Perfect spheres don't touch. Since the circumference of a sphere is a circle, and circles are made of points, and points have no area, then two perfect spheres cannot touch one another, since the point of contact would be nothing plus nothing.

My best stab at a solution is that circles are not really made of points. A point is purely a notional geometrical figure, not a constituent part of something else. So, a sphere can be described as a collection of points, but it clearly isn't such a collection, because then the whole thing would have no area, being a sum of infinite numbers of nothings. But at the same time, the highest point on a circle, and hence on a sphere, is the point that will touch, and it's still only a point...

So an alternative explanation would be that contact doesn't imply area it can be punctilious.

A practical explanation is that any real sphere won't be perfect, it will, even at the molecular level, have flatness. But I don't like that one!

2 1 Equals 0.9 Recurring. This is because 1 is 3 thirds, and a third is 0.3 Recurring, and three times 0.3 Recurring is 0.9 Recurring.

My possible solutions are that fractions and decimals cannot be translated precisely, so a third isn't really 0.3 Recurring; or that it's got something to do with convergence and Zeno's paradox, kind of...

I would be very appreciative of guidance on these.

---

It is surprising how much anxiety can be caused by mathematical notions and the fact that mathematics can never be free of paradoxes. Let's deal with the two touching circles first.

Suppose we have two circles A (radius a) and B (radius B) then we will say that these two circles are touching when the centre of circle A and the centre of circle B are exactly a+b apart. You can think of touching circles as sharing a common point if this will help you to get over the difficulties caused by imagining that a point on one circle touches a point of the other (since this would seem to involve us in an infinite regress).

Next recurring decimals. Its really just a question of how we choose to represent numbers. The exact representation of a third in the decimal system is 0.3 Recurring. It is also possible and it makes most sense to regard 0.9 Recurring as an alternative representation of 1.0. Also 3.4445 Can also be written as 3.44449..., Where... Is understood as indicating an infinitely recurring last digit.

You might like to note that one of the most famous results in modern mathematics, Cantor's proof about the power of the continuum (the number of real numbers between zero and one) explicitly requires us to represent all numbers as infinite decimals in order to avoid any charge of double counting. It is a feature of the decimal system that all numbers can be represented as infinite decimals if we wish to do so.

Shaun Williamson

back

(44) Natalie asked:

I'm 15. I'm sorry if this does not seem a particularly 'philosophical' question, but I feel it is one that can only be answered by someone such as your good self.

As a 'teenager' who has begun to realise that mainstream culture and its values are not for her, instead wanting to improve her knowledge and understanding of the world, it has dawned on me (looking at this site) that I don't know where to begin when it comes to reading philosophy.

A lot of the ideas explored I do not understand, but don't know what to read first I started with Plato's Republic but of course, I did not understand many of the key ideas, and then went to the other extreme of a book which tried to incorporate philosophy into the Simpsons television series a painful attempt to say the least.

So, is there a starting place for an understanding of philosophy which you could recommend, and also, (just out of curiosity), how old were you when your interest began in the subject?

---

With regard to academic philosophy I still think a good start is to read Bertrand Russell's one volume 'History of Western Philosophy'. It may not be the most accurate history of philosophy but it will give you some idea of the complications and breath of the subject.

Philosophy is difficult and you shouldn't expect to understand it all just at the first reading. It takes time.

However since your concern is with society and values you should not ignore two other sources of thought about life I.e. Novels and poetry. For novels that are philosophical in nature read works by Albert Camus and Jean Paul Sartre. However there is a vast wealth of other writers such as Thomas Hardy and Jean Rhys who express important ideas about life and values so don't just think about philosophy. Also read all the novels by Franz Kafka, they will give you some very different and strange ideas. Poetry is more difficult. Try starting by reading the collected poems of W.H.Auden, a modern and accessible English poet who had lots of ideas, about life, love and politics, and he can also be entertaining at the same time.

I first became interested in philosophy at the age of thirteen when I read a book by a Frenchman, Pierre De Latille, titled 'Thinking by Machine'. This was a book about the possibility of creating intelligent thinking machines. I had been brought up as a Christian with the idea that man is different from other animals because he has a soul, so the idea that one day we might create intelligent thinking machines (without a soul) challenged everything that I had been taught to think.

Shaun Williamson

back

(45) AikHui asked:

What is postmodernism and how does it differ from modernism? Is postmodernism going to take over modernism?

---

The three usual historical periods of philosophy are Ancient, Mediaeval, and Modern. The Ancient period consisted of Greek and Roman philosophy and ended with Plotinus. The Mediaeval period began with St Augustine and ended with Scholastic philosophy. And the modern period began with Descartes (or, some say, with Hobbes) and lasts into the present. Recently some philosophers, with quite extraordinary hubris, have taken to calling themselves postern-moderns, in the belief that their contributions to the history of ideas are sufficiently important to warrant this. They are, in my opinion, not even philosophers.

Helier Robinson

back

(46) Mike asked:

Is there a philosophical standpoint which says 'I am the only conscious being in the universe'. My physics students were interested in this idea. They (and I) argued that since each of us can only say for sure that we exist (are conscious) we will never have any evidence that anyone else has consciousness. We argued that for all we know the rest of the 'human race' are merely automatons playing a part in some giant version of the Truman Show!

Hope you can help

If I don't get an answer I will assume that I was right all along and you have been prevented from telling me in order to preserve my delusion!

---

The philosophical view you are talking about is known as 'solipsism'.

However, it is necessary to distinguish philosophical solipsism from the claim — improbable but still possibly true — that despite appearances you are, in fact, the only living being in the universe.

For more on this, see my answer to Tobias.

Geoffrey Klempner

back

(47) Alex asked:

Hello, I have reached a conclusion that is quite dangerous to my health and could lead to a lot of trouble. I need to ask someone and see if they come to the same conclusion. My question is: are you the same person you were 1 year ago or even 5 minutes ago?

I figured that the self changes over time, regarding both personality and physical appearance. As you gain knowledge and change your opinion, your personality changes and you seem to be totally different then you were before. Your physical appearance also changes over time, the cells in your body completely replace themselves in about 7 years (i think). Although your memory really doesn't change over time, only how you perceive this memory does, and how you perceive the world around you. To further define my question: because we are constantly changing and are becoming a new person (except for our memory which ties our life together and gives us the illusion that we are the same person) should I be living completely in the present and totally disregarding the past and future?

You can see what problems this creates. Thanks you.

---

Although I don't agree with your argument that personal identity is an illusion, we can consider what would be the rational course of action for someone who is convinced by this argument.

Derek Parfit in his book 'Reasons and Persons' takes this view of personal identity. In Parfit's view, there is psychological continuity, which decreases over time as our memories fade, but nothing that deserves the title 'identity'.

The importance that we place on identity is therefore, in Parfit's view, misplaced. However, Parfit draws completely the opposite conclusion from the one which seems to be worry you.

So long as I believe in my identity in terms of a special, 'metaphysical' relation between my future mental states and my present mental state, I will seem to have special reasons for giving my future desires preference over the desires of other persons. By contrast, on the view that there is no such thing as personal identity, 'my' future desires and the desires of other persons are on the same level.

It follows that the rational course of action is to always govern one's actions by the rule of maximizing desire satisfaction for all future selves, counting each of one's own future selves as one, and only one amongst the multitude equally deserving future selves.

In other words, far from abandoning yourself to 'living in the present' you ought to adopt a strictly selfless code of behaviour, and work for the betterment of all humankind.

Geoffrey Klempner

back

(48) Jeff asked:

There is a law that states the easiest explanation is the simplest. I think its called Achbar's razor?

---

You are probably thinking of Occam's Razor, which states: Do not multiply entities beyond necessity. This means that you should not invent more theoretical entities than are needed to explain the empirical facts. An equivalent maxim in science is called the Principle of Parsimony of Hypothesis. A third way of saying this is the KISS principle: keep it simple, Simon. Generally the easiest explanation is the simplest, but not necessarily thereby the best; for example, the easiest and simplest explanation of day and night is that the Earth is fixed at the centre of the Universe, and the Sun goes around it.

Helier Robinson

back

(49) Miles asked:

In your (analytical) opinion, is 'free will' possible?

I would have thought that it is a contradiction in terms, and therefore a logical impossibility. Here is my reasoning:

If Causality applies, like causes must produce like effects, so in a precise given situation, only one action/outcome is possible.

If causality does not universally apply, then any effects, actions, and outcomes which are not caused must be random (not the equivalent of rolling a dice, the outcome of which is caused, but 'true random').

If an action/decision is random, it cannot be the perpetrator's wilful choice (since to be wilful implies that it is caused by someone's will, or at least caused by something about the person)

Freedom implies that in a precise situation, more than one action retains some possibility of occurring, I.e. The action is not caused. Will implies that the action is caused. Are the two not mutually exclusive?

Is my reasoning valid?

Have I made any errors of definition?

Thanks for your time.

---

Yes, your reasoning is basically valid. I would restate your last paragraph as: a free decision to act is either caused, or not; if it's caused it is not free, and if it's not caused then its then it's not a decision. The reason that very few people agree with this is that we all have a very strong feeling that we are free to choose. Your argument requires that this feeling is an illusion.

Helier Robinson

back

(50) Paul asked:

I have two paradoxes which I am trying to disprove. They were both given me by a maths teacher in the early 1980s.

1 Perfect spheres don't touch. Since the circumference of a sphere is a circle, and circles are made of points, and points have no area, then two perfect spheres cannot touch one another, since the point of contact would be nothing plus nothing.

My best stab at a solution is that circles are not really made of points. A point is purely a notional geometrical figure, not a constituent part of something else. So, a sphere can be described as a collection of points, but it clearly isn't such a collection, because then the whole thing would have no area, being a sum of infinite numbers of nothings. But at the same time, the highest point on a circle, and hence on a sphere, is the point that will touch, and it's still only a point...

So an alternative explanation would be that contact doesn't imply area it can be punctilious.

A practical explanation is that any real sphere won't be perfect, it will, even at the molecular level, have flatness. But I don't like that one!

2 1 Equals 0.9 Recurring. This is because 1 is 3 thirds, and a third is 0.3 Recurring, and three times 0.3 Recurring is 0.9 Recurring.

My possible solutions are that fractions and decimals cannot be translated precisely, so a third isn't really 0.3 Recurring; or that it's got something to do with convergence and Zeno's paradox, kind of...

I would be very appreciative of guidance on these.

---

One thing to note is that a point is not nothing. Generally your two paradoxes arise from trouble with infinity — so you are right to bring in Zeno. A point is the result of infinite divisibility of a length, and 0.9 Recurring is the result of an infinite multiplication (3 times 0.3 Recurring). My own view (not many will agree with it) is that infinity is an incoherent concept which will always lead to paradox in the end.

Helier Robinson

back

(51) Max asked:

I am a student at San Diego State University and I would like some help with a philosophy question please.

In Descartes 'Meditation', He makes the claim that we could be dreaming, and 'that there are no definite signs to distinguish being awake from being asleep.' Are we dreaming right now? Answer, according to Descartes, yes we could be dreaming. Can a test be performed that will prove that a person is not dreaming right now, if so what is that test?

---

No, there is no definitive test to prove that you are awake. The best that we can do is to be reasonably sure, on the grounds that dreams are much less coherent than waking life.

Helier Robinson

back

(52) Mahin asked:

What is the difference between analysis and speculation?

How it is used and had been using in philosophy?

---

Speculation may be called synthesis, in opposition to analysis. Analysis consists of enquiring into what you already have, while synthesis consists of inventing something new — usually in order to explain the content of your analysis. For the past century or so almost all philosophers have believed that philosophy should consist of analysis only: speculation must be disallowed. The reason why I think that this is wrong is that philosophy must harmonise with science, and science has a very successful synthetic side, as well as a very analytical side. Analysis is science occurs with empirical science and synthesis with theoretical science. Theoretical science consists of inventing explanations of empirical behaviour — explanation by describing 'underlying causes,' it is said. To describe causes is to explain their effects, so theoretical science explains empirical science. 'Underlying' here means non-empirical, or imperceptible, which is what the theoretical is.

Helier Robinson

back

(53) JP asked:

What makes a chair, chair?

Not just in its legs

not just in its color

---

How about its function, what it is made for and used for?

Helier Robinson

back

(54) Nishta asked:

Should there be a complete freedom of speech?

---

I don't think this will ever be possible. In most countries speech which incites people to commit illegal acts will be illegal and this is needed in order to make the legal system consistent. It would be strange if murder was illegal but incitement to commit murder wasn't. Also evidence for conspiracy to commit a crime will often be in the form of evidence of conversations between individuals. Again it would be strange to make an action illegal and not make conspiracy to commit that action illegal. Unfortunately 'freedom of speech' has become a slogan that stops people from thinking about the much more important idea of freedom. Many groups who preach hate and want to take away our freedoms attempt to hide under the 'freedom of speech' slogan.

Shaun Williamson

back

(55) Gary asked:

Using a video monitor with say a 1000x1000 pixels I can easily generate all possible images on this screen with a short bit add program. Starting with all zeros for the binary white screen and adding 1 in binary with each iteration all combinations are produced ending with the binary number with all 1's for black (use 32 bits per pixel to give good color rendition). Once this occurs everything repeats. In this set of unique screen images are any images that can be captured by digital camera. The pages in all possible books, fiction, or non fiction, in all possible written languages as well as mostly random patterns. The point being that eternal repetition occurs if the program runs uninterrupted (this will take a very very long time with present day computers). The set of different images although finite, boggle the mind. This idea applies to a quantum interpretation of reality as well. Now take the subset of images in the pixel set that represent the Mandelbrot set. In the actual mathematical set of Mandelbrot images there is the capability to zoom each pixel to the size of the screen over and over again and the sequence of images are truly infinite in many of the zooms as are the sequence of digits in any transcendental number such as pie or e. This is not the case for the Mandelbrot images in the pixel set of images, although we can always find a match to the sequence image. In real life sequence is important and can lead to unbounded expression as in pie or e. Comments please.

Gary Ehlenberger
Retired Staff Scientist
Member of the Technical Staff
Motorola.

---

I am not sure what the question is. However, there is a Nietzschean take on this. Nietzsche hypothesised that the actual universe consists in a finite number of material bits, governed by the law of causality.

Given this assumption, he believed, it follows that given enough time the total material configuration of the universe must at some point recur. The entire history of the universe will then be repeated from that time onwards, then again onto infinity. Nietzsche called this the 'eternal recurrence'. The doctrine was not his own but was initially espoused by the Stoics.

Nietzsche's argument contains a fallacy. Even if we set a minimum finite size for material bits, provided that we allow infinite variability in spatial position there are possible universes where we know in advance that the total material configuration will never be repeated.

For example, consider a universe consisting of just two concentric wheels, one revolving at speed 2x for any x, and the other revolving at 3x. The initial position, whatever it may be will never be repeated because no multiple of 2 is divisible by 3 or vice versa.

In other words, there is no limit to variety so long as you allow infinite variability in spatial position. If on the other hand, spatial position is quantized, then it is possible that Nietzsche is right.

Geoffrey Klempner

back

(56) Guy asked:

What is the nature of change?

In the literature, I see some people claim that time does not exist. They say that it's all really about change. Then I see others say that change does not exist. They say that's it's all about causality. Yet another group talks about a block universe which is completely spelled out. But they do not address what is going on when we see our own perspective travelling somehow through such a predestined block object. So it appears to me that people find solace in changing the vocabulary they use. But nowhere have I seen any attempt at explaining what it means to say that existence unfolds be it through the vocabulary of time, change, or causality. What does it mean that things exist in one configuration 'and then' in another a moment later. Can someone please address for me any ideas they might have about the nature of this 'and then'. What is the nature of change? Surely there are clever ideas around this. But I have not been able to locate them.

---

The problem that you are grappling with is known as, 'the reality of temporal becoming.'

One way to appreciate the reality of temporal becoming is in terms of a claim about perspective: the block universe, where time is seen as a fourth dimension analogous to space, cannot be our universe because any description we give is from a temporal perspective.

This claim seems intuitively plausible: the problem is that it is impossible to state, in language, that we occupy a particular perspective in time.

Suppose I write, 'I am now responding to a question about time.' Reading this tomorrow, I will interpret this as meaning, 'GK wrote the words, 'I am now responding to a question about time' while responding to a question about time', or 'GK wrote the words, 'I am now responding to a question about time,' at 2.54 Pm GMT on 31.12.07.' The truth of these statements is independent of the fact of what time it is. They will be no less true in a thousand years time, than they are true now.

The idea of the reality of temporal becoming is the idea that there is a fact, which is the fact of 'where we actually are in time', which can only be known at that time and cannot be put into words. This is paradoxical, to say the least.

A similar paradox is involved with the idea that I am writing these words. From your point of view, the statement, 'I am writing these words,' means, simply, 'GK is writing these words.' The 'I-ness of I', like the reality of temporal becoming, can only be appreciated from a particular perspective and cannot be expressed in language.

For more on this question, see my book 'Naive Metaphysics', which can be downloaded from the Pathways site at http://philosophypathways.com/download.html.

Geoffrey Klempner

back

(57) Jenna asked:

In pragmatism, the test of truth of an idea or propositions lies in what?

---

Pragmatism rejects the method of definition as method of clarifying the meaning of ideas or abstract terms, because when trying to give a definition of a universal term, one has to enumerate all the universal predicates of a term, each of which is more abstracted and general than the term defined. Apart from that, the process can give rise to scepticism, for it can go endlessly (infinite regress), unless we stop at ideas such as Pure Being, Substance, Agency etc. For example, if we attempt to define the term 'man', we can first define it as 'a rational animal', but then we have to give the definition of 'rational animal', let's say 'a part of the class of sensitive living things', which in turn can be defined as 'a part of the class of animate bodies', then we can further define it as 'a part of the class of corporeal substances' and so on. Charles S. Peirce the originator of Pragmatism characterizes the process as follows:

'(One word) will be defined by other words, and they by still others, without any real conception ever reached' (Collected Papers of Charles S. Peirce, CP 5.423)

To put an end to this regress, Peirce introduced the pragmatic meaning of ideas, which is referring to their practical consideration. This means that since the essence of anything is a sum of habits that involves (Piercean Category Thirdness) and is directly perceived in our experience, then the meaning of the ideas can be translated in terms of concrete experience and action. In other words the meaning of a sign lies in considering its practical consequences, namely if one exerts a certain action of volition or activity, one will undergo in return certain inevitable perceptions. For example, from a proposition 'honey is sweet' we can deduce the conclusion 'honey possesses sweetness'. Since 'sweetness' can be applied to many things (sweet fruit, sweet chocolate etc.), Is a universal concept of quality therefore is real, for all the sweet things share it in common independently of our mental constructions. The proper way to discover what the abstract concept (symbol) 'sweetness' means, is to translate it into a certain mode of action, which means 'if I put some honey on my tongue, I will experience that is sweet'. Or else, 'diamond is hard', means 'diamond possesses hardness' and the practical consideration of the symbol of 'hardness' is: 'if I press a knife-edge against it I will experience resistance'. To use Peirce's terminology, the dynamical, logical interpretant of the sign is the pragmatic meaning of the idea that the sign represents. To sum up, the practical consideration of a universal quality (sweetness, heaviness, hardness etc.) Consists in the following premise:

'If I conduct myself in a manner x, I will have experience y',

So to say it represents a relation between volition (action with deliberation) and perception; therefore the meaning of the conception of a quality or else its rational purport (CP 5.412) Lies in its conceived effects. Peirce defines the meaning of ideas and the practical consideration as follows:

'In order to ascertain the meaning of an intellectual conception, one should consider what practical consequences might conceivably result by necessity from the truth of that conception; and the sum of this consequences will constitute the entire meaning of the conception' (CP 5.9. 1905)

'All reasoning turn upon the idea that if one exerts certain kind of volition, one will undergo certain compulsory perceptions. Now this sort of consideration, namely that certain lines of conduct will entail certain kinds of inevitable experiences, is what is called practical consideration' (Ibid)

This process of discovering the meaning of abstract concepts through their practical consideration, which Peirce calls pragmatic maxim, can be applied certainly on the certain kind of signs, namely the symbols. The justification of the pragmatic meaning of ideas lies in the reality of the universals (in Aristotelian sense), namely since the universal qualities are real and are not mental creations, for they exist in things as habits or dispositions or behaviours (potentialities), therefore their meaning can be translated pragmatically into terms, which refer to concrete perceptual occasions. As Peirce was influenced by scholastic realism (Duns Scotus) holds that the universals (laws, thirds) are real, exist potentially in the particulars, and are not mental constructions as nominalists argue.

'The property, the character, the predicate, hardness, is not invented by men, as the word is, but is really and truly in the hard things, and is one in them all, as a description of habit, disposition or behaviour' (CP 1.27. 1909)

In order to establish the inferential relation between action and perceived experience Peirce uses the analogue of relation between antecedent and consequent. As he argues, in each syllogism there is an antecedent premise, a consequent premise and the logical consequence, which is the assertion that the consequent follows from the antecedent, in other words the consequence is the relation between them (CP 4.45). By analogue practical consideration is a relation between action and perceived experience (if I conduct myself in manner x, then I will experience y) (1). This practical consideration, Peirce uses in the deductive phase of scientific inquiry by drawing the practical consequences of a hypothesis. For one can easily infer from the proposition (1) the following:

'If I want to have experience y, then I will conduct myself in manner x'.

This means that after the abductive inference and the generation of a hypothesis, one can guess what sort of experiments should conduct, in order to discover if his hypothesis is true (See more about abductive and deductive phase of inquiry) in my answer (88) Ask a Philosopher: Questions and Answers 37.

Finally, apart from the cognitive aspect of the pragmatic meaning of ideas, which consists in their practical consideration, there is also a purposive aspect involved, for one has in mind the purpose of acting in a certain way and attaining a certain experience. This purposive aspect that involves volition (action with deliberation) Peirce calls intellectual purport, and is inseparable element of the pragmatic meaning of ideas. For according to Peirce's definition of his new theory (pragmatism) in 1905:

'The most striking feature of the new theory (pragmatism) was its recognition of an inseparable connection between rational cognition and rational purpose (CP 5.412)

Nikolaos Bakalis

back

(58) Yanyan asked:

What is Aristotle's argument that the 'good of man is an activity of the soul in accordance with virtue'?

---

Aristotle in his Nichomachean Ethics I divides the goods into three classes, the external goods, those that relate to the body, and those to the soul. As all things have a function or activity, the good (agathon) resides in their good function. For example, the function of a lyre player is to play the lyre, his good is to play well, and as a result the virtue or else the excellence (arete) of the lyre player is the good performance of that action. Since life is alertness and activity, the work of the soul and the work of the virtue is to produce good life. The work then of the complete virtue is happiness. As a result, the complete good is a rational activity of the soul in conformity with the best and most complete virtue. For, what distinguishes the virtue is the action, since the deeds are the ones that can be blamed or praised, as they prove the understanding or no of the virtue. On the other hand, when some one is asleep, neither can we give an opinion of his virtue, nor to mark him as a happy man. Therefore he concludes:

'Human good turns out to be activity of soul in conformity with virtue, and if there are more than one virtue, in conformity with the best and most complete.' (Aristotle, Nicomachean Ethics I, 1098 a, 15-17).

As he proceeds his study on virtue he concludes in the end of his Nichomachean Ethics X that happiness is an activity in accordance with virtue, therefore it should be in accordance with the highest virtue, which is proper to the best part within us. At this point Aristotle meets Plato's view, namely that the pleasure of the best part of the soul is the highest, and therefore it is happiness. Since the best part in us is the intellect (nous), for it is the divine and noble element within us, then it follows that the activity in accordance with its proper virtue will be the highest pleasure, namely happiness. This activity of intellect is speculative and theoretical (theorein: contemplating), because this is the actuality (entelecheia) of the intellect.

'If happiness is activity in accordance with virtue, it is reasonable that it should be in accordance with the highest virtue; and will be that of the best thing in us. Whether it be intellect or something else that is this element which is thought to be our natural ruler and guide and to take thoughts of things noble and divine, whether it be itself also divine or only the most divine element in us, the activity of this in accordance with its proper virtue will be complete happiness. That this activity is speculative we have already said.' (Aristotle, Nicomachean Ethics X, 1177 a, 10-23).

Excerpts are from my book 'Handbook of Greek Philosophy'

Nikolaos Bakalis

back

(59) Clare asked:

How are thinking and language related?

---

There is no simple answer to your questions. The concept of thinking has many different uses here are some of them.

1. X thought that he wanted to go home.
2. X thought 'I want to go home'.
3. X thought intensely about the remaining problems of quantum mechanics for the next six months.
4. X said 'I want to go home'.

For 4 to be true X must possess a language even if it is a sign language like that used by deaf people (then we would write 'X signed 'I want to go home' '). Notice how different 1,2 and 3 are even though they all involve the phrase X thought. If 1 is true then it does not imply that 2 is true. However if 2 is true then it usually implies that 1 is true.

One thing that 1,2 and 3 have in common is that it would not make sense to assert any of them about a particular individual unless that individual knew how to use a language. So asserting 1 or 2 about a particular individual implies that he has the capability to utter or at least to understand 4.

So it makes no sense to assert 1,2,or 3 about a dog for example because dogs don't possess the capability of speaking or understanding a language except in a fairy story but of course in a story, dogs trees or even stones can speak. In real life they can't and don't even if dog lovers think they do.

However don't make the mistake of thinking that there is any simple formula that can express precisely the connection between these two things.

Shaun Williamson

back

(60) David asked:

Why do philosophers always seem to wear beards?

---

As far as I know a majority of philosophers do not wear beards. However it is possible that a higher proportion of philosophers have beards compared to other occupations. I would guess that the reason for this is that they don't want to waste time shaving.

Shaun Williamson

back

(61) Rachel asked:

I am coming really dumb...Purely ignorant about any of its fact! I grew up with hindi movies and the tale of influencing baby's appearance in the womb by looking at the picture of mother's favourite personality so that the baby will be born with similar appearance as the individual looked at whilst pregnant. Does it have any relevance or is it just a fabricated tale? So then if I look at a beautiful celebrity like Angelina Jolie would by baby be born looking similar in appearance as Angelina Jolie ?

---

Yes you are wasting your time. Your baby's appearance will be determined by your dna and your husband's dna and also by your parents' and his parents' dna.

Beyond that your health during pregnancy is important as is the child's diet during the early years of life.

Shaun Williamson

back

(62) Angel asked:

Why there is a night and day?

---

Massa asked:

First, what can we make of existentialism and the fact that it was one of the dominant philosophies of recent times? Are we to conclude, with sartre, that we live in a world without objective values a world with no meaning in which we must take responsibility for 'creating our own essence'? What are the implications, both positive and negative, of such a philosophy?

Second, are there, after all, eternal and objective values as socrates thinks and as sartre denies? What is the role of knowledge, belief, and even faith in our sorting out of the world and its meaning for us personally? What, in the end, is the value and importance of philosophy, in your view? Please help me with this two question.

---

Sartre does not conclude that we live in a world without objective values. Instead existentialism starts with the view (following Hume) that philosophy has failed to provide a rational foundation for objective moral values.

Existentialism promotes concepts such as commitment and authenticity etc. However existentialism can not help us to decide whether we should be a committed Nazi or a committed socialist or a committed Christian. This is the chief defect of existentialism, it is morally empty. It cannot tell you what you should do, it can merely suggest poses you should adopt in the face of a morally empty universe. You are supposed to adopt the pose that suits you best.

Promoting the idea that there are no moral values is in itself an evil and immoral idea. However existentialists (and Hume) are correct when they assert that philosophy has failed to provide a rational foundation for ethics. What they overlooked is that morality does not need such a foundation and that is why philosophy cannot provide it with one.

Shaun Williamson

back

(63) Adam asked:

What would be the qualities of creatures that necessarily did not exist?

---

A 'creature' in the literal sense is a 'created thing' and therefore exists. It follows that there could not, logically, be a creature which necessarily does not exist.

However, we may define an entity which necessarily does not exist as the satisfier of any description which is self-contradictory. For any self-contradictory description F and any x, it is necessarily not the case that there exists an x such that F(x).

This definition may be expanded, as appropriate, if you hold that there can be necessity (and corresponding impossibility) which is not reducible to logical necessity/ impossibility: e.g. The description, 'all red and all green' as applied to a visible surface.

Geoffrey Klempner

back

(64) Pavel asked:

1: What are we?

2: What is death?

I've thought about this for a very long time...

But eventually, I come only to the conclusion that death does not exist. For we are, what we percept, and what we remember.

Each time we forget, a part of us dies. So whilst going through life, we change; we die, and are reborn.

So when death comes, isn't that just a matter of change?

Isn't there any other answer?

---

Your question highlights an important truth about death: that the death of a conscious being cannot be the same as simply physical destruction or ceasing to exist.

If I drop a vase, the vase ceases to exist as a vase. All that is left are the shattered pieces of the vase. If I burn a piece of paper, the paper ceases to exist as a piece of paper. All that is left is a pile of ash.

Similarly, if a human being smashed to pieces or burned, the human being ceases to exist and all that remains are the stuff that human beings are made of, or burnt ashes.

However, this ignores the element of subjectivity. My existence concerns me because I am aware of my own existence. Suppose that my body is smashed or burned: it does not logically follow that there will be no more 'me'. It is logically possible that I will wake up 1000 years in the future in a new body.

This thought, which seems initially to entail some form of mind-body dualism, is in fact fully consistent with materialism. Daniel Dennett speculates, in 'Consciousness Explained', that if you could write the 'program' for the human mind, then there would be nothing to prevent uploading that program into a new body at a future date, long after the original body which ran the program has ceased to exist.

It doesn't matter whether you believe in the technical possibility of 'writing a program for the human mind' or not.

Unfortunately, there is a catch. If we accept that subjectivity has a claim, which cannot be overturned by anything that physically happens to the body, then by the same token we have to allow that there is no distinction between seeming to continue in existence — e.g. Seeming to wake up 1000 years in the future — and coming into existence with a set of false memories.

Following this line of thought one is led to the conclusion that from the purely subjective standpoint, there is no distinction between 'continuing in existence' and 'seeming to continue in existence'.

It follows from this that if we are defining 'death' purely in subjective terms, then 'I' cannot die because the pure 'I' of present awareness does not continue, but is merely replicated from moment to moment. We call such replication 'identity' only because we have the physical body as a relatively constant point of reference.

Geoffrey Klempner

back

(65) Bryce asked:

If Wittgenstein is correct, what problems do you see for Chomsky's argument from linguistic creativity?

---

I probably shouldn't even try to answer your question but its hard to resist. If there is an argument here then its between Chomsky and Malcolm's interpretation of Wittgenstein (and Malcolm completely misunderstood Wittgenstein). Wittgenstein does not offer a theory of what it is to understand a language. He discusses when we say (outside of philosophy) that someone understands a language and compares that to what philosophers say.

At the same time linguistics has moved beyond Chomsky and his attempts to draw metaphysical conclusions from what should simply be a science. Now I know that this won't be a helpful answer but there's nothing I can do about that.

Shaun Williamson

back

(66) Hamed asked:

In The Nature of Morality Gilbert Harman argues that the fundamental problem of ethical judgments is their immunity from observation. We cannot see or observe moral wrongness or rightness.In your view, can the problem Harman identifies be solved?

(A) what is the problem Harman has?

(B) would a an utilitarian agree of disagree with Harman?

(C) your reasoned opinion of the significance of the problem Harman identifies.

---

No, I have not failed to recognise this as an essay question, and no, we do not answer essay questions for students.

Nevertheless, I must point out that in The Nature of Morality Gilbert Harman argues something that is demonstrably false, as demonstrably false as anything ever can be. If the liberators of Belsen did not observe 'moral wrongness', then I do not know how we can describe their experience accurately.

What Harman seems to have here is a theory. He may find ways of mangling or redescribing our experiences in order to sustain internal coherence in that theory, but such achievement does not attract me to his proposal.

Further, it is typical of a certain approach to ethics to speak of 'moral wrongness', as if there were wrongness plain and simple (as in for example in maths, perhaps), and 'moral wrongness' as an additional and stranger, less comprehensible attribute. The suggestion contained in such language is false. It is quite as plausible that rightness in maths is moral (as Plato and Simone Weil maintained), as that words like 'good' 'bad' 'right' 'wrong' 'ugly' 'beautiful' etc have always two entirely distinct senses, the moral and the non moral. Again, this second claim is a bizarre account upheld only in order to save the internal coherence of an unattractive theory, the chief merit of which is supposed to be that it tells us what the 'problem' with moral judgements is. There is no special problem of moral judgements. They are perfectly alright as they are.

David Robjant

back

(67) Iain asked:

Has the private language argument advanced by Wittgenstein as a refutation of empiricist theories of mind + meaning itself been refuted ?;

---

I am not sure that your summation of the import of W's reflections on the impossibility of a private language is correct. In any case, if one wishes to refute what the later W has to say, which in my view in fact amounts to an empiricist theory of mind and meaning (albeit one dramatically at odds with all previous such theories), it may be fruitful to consider how W approaches the matter. His wonderment is directed at individual utterances, and at the problem of how several instances of an utterance might be connected — what, if anything, underlies my use of 'red? — The answer being that nothing private underlies it, and that our linguistic practice with 'red' is all the connection there could ever be between this cry 'red!' And that cry 'red!'.

To some extent, W resembles Hume, who, supposing that human experience consisted of a series of distinct packets of data, wondered how those packets might be connected, what might lie beyond them, how there can be an idea of causation, and so on. Both end up saying that all there is, at bottom, is habit — though W importantly varies the notion of habit by invoking the linguistic community rather than the individual as the relevant bearer of habit, or 'practice'. Hume and W both, I believe, suffer from their starting point. If we begin by treating the particular as unproblematic, it ought not to surprise us that we end by making the idea of a universal problematic. These ends are contained in their beginnings. The beginning, I think, is wrong. We ought not to wonder about how this and that particular utterance of the same word might be connected, but rather, more fundamentally — and more platonically — about how sameness itself can feature in our experience. Habit and use, being only ways of relating various particular samenesses, will not do the job here.

David Robjant

Gaurav asked:

Is happiness a choice?

---

No. If it were then everyone would be happy.

Helier Robinson

back

(68) Heena asked:

To what extent are languages and thinking related?

---

First of all, we should be clear about thought, which must be distinguished from imagination. Imagination uses images in the mind, and is concrete; thought uses ideas and is abstract. Some philosophers, called nominalists, believe that all thought is silent speech: there is no thought without language, or words are counters of the mind. Nominalists have trouble explaining synonyms, which being different words must have different meanings, and translation from one language to another, since different languages must have different meanings. Another group of philosophers, called conceptualists, claim that thought operates with abstract ideas; but they do not say what an abstract idea is. A third group called platonists, or realists, accepts abstract ideas but go further and claim that each abstract idea is a copy of a real abstraction.

For example you and I each have an abstract idea of the number two, and these abstract ideas are ideas of the real number two. I suspect that nominalists are people with very vivid imaginations and because of this they cannot discover abstract ideas introspectively, much as we cannot see the stars in sunlight, while conceptualists have weak or no imaginations and so can discover abstract ideas. So, to answer your question, if you are a nominalist then language and thought are the same thing insofar as it is abstract. But if you are a conceptualist then language and thought are distinct. You can have a word joined to an abstract idea, and this is a concept; and you may think with concepts; or with abstract ideas alone, as when a thought is 'on the tip of your tongue' or when you are thinking up original ideas for which there are not yet names; or you may think with words alone, as when using words algorithmically — that is, according to set rules, as in long multiplication.

Helier Robinson

back

(69) John asked:

I see life in terms of energy. Everything I can perceive currently I can relate to in terms of energy and the way it flows around. Maybe even your life force or your time on this earth can be seen as a store of potential energy. I like to believe that myself as a 'being' is a manipulator of energy, a lot better than you first might think as well.

I feel my human life is indeed a gift but on the most part I am spoiling it with my biology, ego and social conditioning. In plain sometimes I think I am on the most part living a blind illusion. In my search for Wisdom and to understand the 'reality' I have followed paths of dreams, meditation and intoxication, they all seem to lead me further down the same spiritual path. I am more than this body and feeding off the essence of Zen finding our way out of this illusion is the ultimate spiritual/intellectual/emotional challenge of life. Leading a wise life and preparing for death.

The other side of me though, the right/wrong driven reasonably well educated boy is scared that I am just separating myself from the population and the world that was created around me. I know I am falling out with the rest of the world and pretty fast. Wars, racism, the world economy, Commercialism, I despise them all.

Is there any insight you can offer or comment on this predicament I seem to have found myself in? Simple logic or some guidance would be great to take on board.

---

First of all, you might do better to think of yourself as a manipulator of negative entropy rather than of energy. Negative entropy may be thought of as a measure of order and complexity. The physicist Erwin Schroedinger defined a living organism as a system of very high negative entropy in dynamic equilibrium. In terms of this, spiritual advancement may be thought of as increase of negative entropy of the mind, and it seems that this is what you are seeking. How you go about it is the main problem: Zen might do, or joining a monastery, or meditation, or prayer. There are lots of teachers out there, many contradicting each other, and lots of organisations, but in fact you have to find your own way. My book 'Belief Shock' might help a bit: you can download it from the web at http://www.sharebooks.ca.

Helier Robinson

back

(70) Yeshua asked:

What came first the chicken or the egg?

---

The egg, of course. Chickens evolved from dinosaurs, which laid eggs.

Helier Robinson

back

(71) Gianni asked:

If God and Superman fought, who would win ?

---

This is a variant of the question, 'Can God make a stone which is too heavy for Him to lift?'

Assume that 'Superman' is created by God to be invulnerable and incapable of being defeated in a fair fight. Then even God cannot defeat Superman.

The answer to this riddle is that the assumption I have just made contains a hidden contradiction. It is no more possible for God to create 'Superman', thus defined, then it is possible for God to create any other logically impossible object, e.g. A four-sided triangle.

Geoffrey Klempner

back

(72) Joseph asked:

Has Western philosophy, (beginning approximately with the preSocratics), ever had a tradition of specific practices or exercises (similar to practices in Eastern philosophy and religion, such as meditation), passed down directly from teacher to student? (I am excepting, of course the practices of argument, dialogue, and reading)

I have read that there is evidence that there was a very strong tradition of meditative and other exercises, beginning with Socrates, but that these were never written down, and gradually declined after the time of Plotinus, finally disappearing entirely from academic philosophy.But I have not been able to find the sources of this evidence.

---

The answer to your question is, Yes.

See: Pierre Hadot 'Philosophy as a Way of Life' Edited and with an Introduction by Arnold I. Davidson. Oxford, Blackwell 1995.

Geoffrey Klempner

back

(73) Guy asked:

What is the nature of change?

In the literature, I see some people claim that time does not exist. They say that it's all really about change. Then I see others say that change does not exist. They say that's it's all about causality. Yet another group talks about a block universe which is completely spelled out. But they do not address what is going on when we see our own perspective travelling somehow through such a predestined block object. So it appears to me that people find solace in changing the vocabulary they use. But nowhere have I seen any attempt at explaining what it means to say that existence unfolds be it through the vocabulary of time, change, or causality. What does it mean that things exist in one configuration 'and then' in another a moment later. Can someone please address for me any ideas they might have about the nature of this 'and then'. What is the nature of change? Surely there are clever ideas around this. But I have not been able to locate them.

---

The best definition of change that I know of is that a change is a dissimilarity in parallel with a duration. If you do not have both a dissimilarity and a duration, there is no change. Causation is something else: given a cause, its effect is a singular possibility — the only possibility (as opposed to a plural possibility, which is a contingency, and a zero possibility, which is an impossibility.) Usually, of course, causes produce changes and changes are caused. Many people have trouble with this because dissimilarities, durations, and possibilities are all relations and a lot of people have trouble thinking about relations: what do they look like, do they exist, etc.

Helier Robinson

back

(74) Adam asked:

What would be the qualities of creatures that necessarily did not exist?

Something does not exist, necessarily, if it has contradictory qualities — such as spherical and not spherical. Any pair of contradictory qualities guarantees non-existence.

Helier Robinson

back

(75) Hare asked:

Why a valid syllogism in figure 2 must have a negative premise?

---

In figure 2 the premises are P--M and S--M. If M is undistributed in each, the fallacy of undistributed middle occurs. In both the affirmative propositions, SaP and SiP, P is undistributed; while in both the negative propositions SeP and SoP, P is distributed. So to avoid undistributed middle, one of the premises must be negative. Also, only one must be negative, since no valid syllogisms have two negative premises.

Helier Robinson

back

(76) David asked:

It is asserted that humans evolved though millions of years from ape like creatures to today's human design. If this is true, why is it that in the handful of cases where we find these children that were left alone in the wild to be raised by wolves or other type of animal besides humans, it is impossible to teach them human like behaviors, even after years of human intervention after being discovered?

---

It is not just asserted that humans evolved from ape like creatures. All the scientific evidence we have (and we have a lot of evidence) shows that humans evolved from ape like creatures. If you want to doubt that this is true then you would need very good evidence indeed to make it sensible to do so.

With regard to the wild child phenomenon, we know that certain sorts of learning require human interaction at a very early age. An example of this is acquiring a language and many of the things we can do as humans depend upon understanding a language. If a child does not have interaction with other humans for the first few years of its life then the parts of the brain that deal with language no longer function correctly. In a similar way a chimpanzee deprived of contact with its own kind at an early age may never be able to fit into chimpanzee society later on.

Shaun Williamson

back

(77) Mike asked:

If there is no God, can there be any absolute? Can there be evil? Who gets to determine what is evil? In small closed societies (like the mob) wouldn't what others call evil be good and vice versa?

---

The idea of good and evil is logically prior to the idea of God. Even if you believe in God you have to decide if God is good or evil before you decide to worship him.

How do we determine what blue is. If there is no God who gets to determine what blue is? It is determined by the common agreement of human beings. Now apply the same idea to good and evil. Language is made by humans for human purposes. God doesn't get to decide any of these things and we don't need any absolute.

Shaun Williamson

back

(78) Teri asked:

Why can't the desire for peace and love work ?

---

The desire to be an artist doesn't make you an artist, that requires talent and hard work. In the same way the desire for peace and love doesn't create peace and love. To create these things requires constant unending effort. There is an old saying, 'If wishes were horses, beggars would ride'.

Shaun Williamson

back

(79) Ica asked:

If Janet, in an ordinary game of dice, hopes to throw a six and does so, we do not say that she threw the six 'intentionally'. On the other hand, if Janet puts one live cartridge into a six chamber revolver, spins the revolver as she aims it at Linda and pulls the trigger hoping to kill Linda, we would say if she succeeded that she had killed Linda intentionally. How can this be so, since in both cases the probability desired result is the same?

---

The probability may be the same but this has no relevance to your example. You can throw a dice thousands of time and still fail to throw a six. If you fire a six shot revolver with a bullet in, six times, then you will only fail to fire a bullet into Linda's head if the gun develops a fault.

You are confused here about different uses of the word 'intentionally'. Suppose Frank decides to kill Fred by throwing an ice cube at his head as fast as possible. Frank intends to kill Fred even if he is unaware that you can't kill someone by throwing an ice cube at their head.

However suppose I am aware that you can't kill someone by throwing an ice cube at their head, then even if I intend to kill Fred, I can't intend to kill him by throwing an ice cube at his head.

Suppose I decide to kill Fred only if on my next throw of the dice, I throw a six. I do throw a six and I kill Fred. Is my killing of Fred intentional? Read J.L. Austin's essay 'A Plea for excuses'.

Shaun Williamson

back

(80) Ica asked:

If Janet, in an ordinary game of dice, hopes to throw a six and does so, we do not say that she threw the six 'intentionally'. On the other hand, if Janet puts one live cartridge into a six-chamber revolver, spins the revolver as she aims it at Linda and pulls the trigger hoping to kill Linda, we would say if she succeeded that she had killed Linda intentionally. How can this be so, since in both cases the probability desired result is the same?

---

When Janet puts one live cartridge in a revolver, spins the chamber and pulls the trigger, her intention would be correctly described in the following way, 'If the chamber contains a bullet, then Linda will be shot.'

When Janet throws a die simply hoping for a six, there is no intention until we specify some consequence which will follow from the die falling on a six. E.g. 'If I get a six then I will do XYZ.' In order for there to be an intention, there has to be an element of choice of what Janet will do if the die falls on six.

Geoffrey Klempner

back

(81) Lakpa asked:

Human being is saying that tree is present on earth? What will be the reply of tree? Human being is present on earth or not?

If not then what is the meaning of our see and speaking?

How human being came to know that he is human being? He himself has given the name or by some other thing in the earth system?

Will there be earth after the extinction of human being? If yes, how? And if no, than why we are predicting that there was evolution when there was no human being?

---

The assumption behind your question seems to be that an entity of type XYZ cannot exist unless someone refers to it, e.g. By giving it a name, 'XYZ'.

Thus, human beings only exist because we give ourselves the name, 'human being'. If you asked a tree whether human beings exist, it would not answer because trees do not have language. From a tree's point of view, therefore, human beings do not exist because a tree has no name for them.

Ian Hacking in his book, 'Why Does Language Matter to Philosophy?' Describes an argument he once had with a student who tried to defend this view of naming. When Hacking replied, 'But there are polar bears which exist in the Arctic which no-one has ever referred to!' The student replied, 'You've just referred to them!'

The claim that human beings would not exist if they were not referred to also gets a measure of specious plausibility from the notion that the world in which we find ourselves is initially whole and without divisions of any kind, and it is only language which divides reality up into 'things'. However, this is just a metaphor, a picture, which is intended to convey the idea that there is no absolute necessity in our making the conceptual distinctions that we do.

Suppose we accept the rule, 'Never allow the existence of any entities XYZ unless you refer to them.' How will this affect anything that we want to say? If I assert that human beings evolved from apes, human beings and apes may be said to exist provided that I successfully use the referring expressions, 'human beings' and 'apes'.

In fact, there is nothing that this alleged 'rule' rules out. You can carry on just as before. Suppose paleontologists claim to discover a new species of dinosaur. Once they have described or named it, they can then can legitimately go on to make assertions about what the dinosaur ate, where it lived and so on, millions of years before it was ever given a name.

Geoffrey Klempner

back

(82) Maritza asked:

My brother who lives in mexico went to a philosophy class and they told him that according to philosophy god doesn't exist. Is this true and why?

---

No it isn't true. What they should have told him is this: 1. There are no philosophical truths that all philosophers agree are true I.e. Everything is open to dispute. 2. There are philosophical problems but there are no agreed solutions to them. So after 2500 years of philosophy, philosophers are unable to agree about anything.

All you can say is that according to philosopher Y, X is true. You can never say according to philosophy X is true. There will always be some qualified philosopher who think X is false.

Shaun Williamson

back

(83) Tobias asked:

I am really struggling with the concept of metaphysical solipsism. My whole life could be a fantasy/ dream, and in 'reality' 'I' may be some form of lonely spirit in space.

For a long time I thought it was an interesting explanation of 'everything' as long as I could believe that I am able to experience 'whatever I what to experience'. Of course, some may call the idea insane.

But then I started to almost panic about the idea being 'caught' in a potential endless sequence of ultimately lonely and meaningless illusions.

I don't know if I am suffering from 'solipsism syndrome', but being around many people in public places usually makes me feel better.

My question is, if solipsism is (still) considered a philosophical challenge/problem?

Thank you!

---

There are no agreed solutions to philosophical problems and that includes solipsism. My own opinion is that there is no solution to the problem of solipsism but there is no problem either. You can't solve a problem that doesn't exist. The difficult thing in philosophy is getting to see that the problem doesn't exist.

Shaun Williamson

back

(84) Lakpa asked:

Human being is saying that tree is present on earth? What will be the reply of tree? Human being is present on earth or not?

If not then what is the meaning of our see and speaking?

How human being came to know that he is human being? He himself has given the name or by some other thing in the earth system?

Will there be earth after the extinction of human being? If yes, how? And if no, than why we are predicting that there was evolution when there was no human being?

---

Trees don't talk and they don't have language. Obviously there are human beings on the earth. There is no meaning to our seeing and speaking except the meaning we give to it.

Language was invented by human beings, not by God. We are human beings because that is that name we give to ourselves. Evolution applies to all living things, the living things that existed before man and to any living things that exist after man becomes extinct.

Shaun Williamson

back

(85) Kris asked:

Why do we exist in the world we all live in?

---

Language and therefore questions and answers were invented by humans not by God. One of the consequences of this is that we can ask questions that have no answer and that even God cannot answer. For example we can ask 'What is the greatest prime number?'. This looks like a sensible question but it isn't and it's one that even God cannot answer.

In that spirit I would ask you to consider what sort of answer you expect to your question. Is it a sensible question or is it more like asking why are some things blue and some things red. Why are some things sharp and some things blunt?

Shaun Williamson

back

(86) Paulus asked:

Is mind matter or is matter mind?

---

Philosophers cannot agree on this question. According to the materialists, mind is matter. According to the idealists, matter is mind. According to the neutral monists, there is a 'neutral stuff' out of which both matter and mind grow. Yet another point of view is that the mind/ matter distinction is ultimately indefensible, as shown by the difficulty of defining each of them. There is an old philosophic joke: 'What is matter? — Never mind; what is mind? — No matter.' If you can answer your question to the satisfaction of everyone you will become a famous philosopher.

Helier Robinson

back

(87) David asked:

Considering the fact that their is no evidence at present to confirm God, why do people with above average intelligence believe that God exists? Surely it's not logical thinking?

Some people, of above average intelligence, claim to have had spiritual experiences which satisfy them as to the existence of God. If you have not had such experiences it may mean that there is no God, or it may mean that there is a God but you are blind. You can believe one or the other, but no one can prove one or the other.

Helier Robinson

back

(88) Jason asked:

How can evil exist if I do not think of anything as evil?

An example being...

If a mass murder does not see the taking of a human life as 'evil' then is it? Does good and evil just depend on the opinion of the masses?

---

It could be that evil, like beauty, is in the eye of the beholder. But it could also be that there is an absolute beauty and an absolute evil (to say nothing of absolute truth and absolute goodness), which we apprehend 'through a glass darkly' because of subjectivity and inexperience. If you choose one side or the other, that does not make your choice the true answer.

Helier Robinson

back

(89) Tobias asked:

I am really struggling with the concept of metaphysical solipsism. My whole life could be a fantasy/dream, and in 'reality' 'I' may be some form of lonely spirit in space.

For a long time I thought it was an interesting explanation of 'everything' as long as I could believe that I am able to experience 'whatever I what to experience'. Of course, some may call the idea insane.

But then I started to almost panic about the idea being 'caught' in a potential endless sequence of ultimately

lonely and meaningless illusions.

I don't know if I am suffering from 'solipsism syndrome', but being around many people in public places usually makes me feel better.

My question is, if solipsism is (still) considered a philosophical challenge/ problem?

---

I think that solipsism is still one of the greatest philosophical challenges. However, unlike other philosophers who have taken solipsism seriously, my starting point is not, 'OK, we know that solipsism is false, now how to we prove that?'

My starting point is that I don't know whether solipsism is true or false, because I am not sure what solipsism means.

Firstly, let's get out of the way the frightening possibility that you are in fact the only conscious being in the universe. That is an empirical hypothesis which cannot be ultimately disproved. E.g. A while ago you were secretly hooked you up to a Matrix-like reality simulation, and then there was a cataclysmic event as a result of which all other conscious beings in the universe perished.

But this is not what philosophical solipsism claims. Philosophical solipsism is a metaphysical, not an empirical theory. It puts forward an interpretation of the facts rather than making a factual claim.

Whatever the facts may be, the solipsist argues, all that exists for me is experience. In the world of my possible experience, other 'subjects' can only ever be objects of my perception; I can never perceive another person's subjectivity. That is a line of thought which seems to point towards solipsism as a logical necessity.

I think that this line of reasoning is flawed (for the reasons for this see my answer to Phil) but in order to show this we need to concede a vital point to the solipsist: that there is no way to conceive of 'the world as such' without including the fact that I am one of the persons in it.

That claim in, a way, vindicates Descartes' claim that 'I exist' is the starting point for philosophy. Any 'refutation' of solipsism, only makes sense in relation to the person asking the question: myself.

Geoffrey Klempner

back

(90) Theresa asked:

This is a question of Aristotle's laws of non contradiction.

That I have just discovered.Aristotle 'one cannot say something that it is and that it is not in the same respect and at the same time.

'Its plain that the same thing won't be willing at the same time to do or suffer opposites with respect to the same part and relation to the same thing.'

'Lets look at he statement 'Never say never.'

Meditating on this statement for quite some time now and relation to the to law.

I have discovered many intriguing points in it.

First lets get this thought out of the way.'What is the mathematical sequence of the phrase itself by the code of what lies in the subjective thought of sequence or contradiction within itself.Even though its not an object.And the truth is not in the objective to solve te riddle.One must look deeper

And even though it is separated by a word of 2 different points. It within itself expresses same part and will to the same respect. The person contradicts himself by saying never say never... But still does not fall under the coe of the law of non contradiction because the self and statement are 2 different parts. But if I say something that contradicts itself with the thought of 2 different directions of relation.. It still doesn't fall under the law of non contradiction. But this I see as the same part..Now we have to look at. Is it a logical statement that has value? Is it true. Is it logical to suggest to someone.. I think so... So does this break the laws of Aristotle's law of non contradiction... And is he only objective minded and really not of the higher intellect. Aristotle is intelligent and has an analytical mind. But how limited is it? So does this observation of mine fall under(asking from another point of view )... Proving the laws of non contradiction as incomplete?

---

'Never say 'never'' is not a statement but a command, an example of, 'Don't do X', for some X.

As such it cannot be self-contradictory, in the sense of a statement which is necessarily false, like 'A and not-A'.

However, we can examine the conditions under which the command, 'Never say 'never'' is capable of being obeyed.

I can tell you to 'never say 'never'' but only if I do not conform my actions to the command. It is not possible for the person issuing the command, 'Never say 'never'' to obey that command. I break the command in the very act of asserting it.

Suppose that someone put forward 'Never say 'never'' as an alleged principle of reasoning. For example, you might think that as time is infinite, there is no way, in principle that one can ever say for certain that event E will not occur, for any E which is not self-contradictory. This seems a plausible argument for asserting that principle.

e.g. To give an example from mathematics, unless we can prove that the hypothesis that four consecutive sevens occurs in the expansion of Pi leads to a contradiction, we can never be in a position to say that four sevens never occur in the expansion of Pi. (So far as I know, no-one has yet found four consecutive sevens in the expansion of Pi.)

This comes down to to: 'Never say 'never X', unless you can prove the logical impossibility of X.' Does that command go against itself, in the sense of being incapable of being obeyed by the one who states it?

No, because the first 'never' and the second 'never' have different meanings. The first 'never' has the sense of a command, while the second never has the sense of a negative existential statement: e.g., 'It is not the case the four consecutive sevens occur in the expansion of Pi.'

Geoffrey Klempner

back

(91) Christine asked:

As a newcomer to the subject of philosophy I have many questions. Why are we here? Is an obvious one, but my thoughts go further — are we really here, or just figments of our own perception of reality, and to expand on that, what is reality?

---

Whether other people are really here is moot, but you can be quite certain that you are here, since you have to be in order to ask the question. I don't know what you mean by 'just figments of our own perception of reality' but if we are figments of our own perception then we are here, since we are perceiving. There are two usual philosophical definitions of reality. One is that reality is all that we perceive around us that is non-illusory and the other is that reality is all that exists regardless of whether anyone perceive it or not. Common sense has it that these two definitions are equivalent: all the non-illusory that we perceive around us continues to exist when no one perceives it. There are difficulties with both definitions and with the common sense view. Thus, can you perceive anything that is wholly free of illusion? And if you can, how would you know it to be so? (Illusion are unreal, necessarily.)

Again, if the real exists unperceived, how can we know about it? And if the common sense view is correct then real objects that we perceive around us are composed of sensations, which are manufactured in our nervous systems and thereby dependent on perception and so unreal. One approach to all this is to start with metaphysical principle, on which all philosophers agree, that there are no contradictions in reality; because a contradiction is necessarily false and reality is the basis of truth. So a necessary condition for your understanding of reality is that your thought about it must be free of contradictions. Go ahead from there, and good luck.

Helier Robinson

back

(92) David asked:

It is asserted that humans evolved though millions of years from ape like creatures to today's human design. If this is true, why is it that in the handful of cases where we find these children that were left alone in the wild to be raised by wolves or other type of animal besides humans, it is impossible to teach them human like behaviors, even after years of human intervention after being discovered?

---

The usual explanation is that as a young human grows from infancy it has windows of opportunity for learning, but these windows close after a certain time. Windows such as learning a language, or more than one language, or socialising with peers. Feral children miss out on these windows. Another example is with people who are born with a cast in one eye; this can be corrected surgically, but if it is done too late the person can never learn to see stereoscopically. Similarly some who are born blind but can have the affliction cured in later life, never learn to integrate what they see with their other senses.

Helier Robinson

back

(93) Henderson asked:

How can anything be false when truth is subjective?

---

How do you know that truth is subjective? Ask yourself the questions: 'If 'Truth is subjective' is absolutely true then is it false? And if it is not absolutely true, then is it true?'

Helier Robinson

back

(94) Madtuna asked:

Why does my shit stink?

---

The stink is just a smell that disgusts. Anything that has survival value tends to be incorporated into our genes and in the past all human excretions — shit, piss, vomit, spit, sweat — carried a danger of infection with a disease; so we learned to avoid them; and this learning we experience as disgust. I hope you are not disgusted with this serious reply to your irreverent question.

Helier Robinson

back

(95) George asked:

We believe that when we die all parts of our body, including our brain decompose and eventually turn to dust to join another part of the cycle of life. But where do you think our thoughts go when we die?

---

This is like asking what a flame of a candle looks like after it is blown out, our what a rain drop looks like after it fall into the ocean. When something ceases to exist it does not 'go' anywhere, it just ceases.

Helier Robinson

back

(96) Marlese asked:

A quota is necessary to hire and promote the best qualified candidates because decision makers judgments are tainted with unconscious prejudices against particular groups of people.

How plausible is this argument?

Do you agree with this method of employing someone?

If not, why not?

---

The fundamental objection to affirmative action is that it is discriminatory, and its advocates base their quota position on non-discrimination. But your argument is peculiar: quotas do not hire and promote the best qualified candidates: quotas discriminate against the best qualified. Yes, unconscious prejudices can taint judgments, but that is because no one is perfect; but one still needs to rely on those qualified to judge qualifications.

Helier Robinson

back

(97) Tobias asked:

I am really struggling with the concept of metaphysical solipsism
my whole life could be a fantasy/dream, and in 'reality' 'I' may be some form of lonely spirit in space.

For a long time I thought it was an interesting explanation of 'everything' as long as I could believe that I am able to experience 'whatever I what to experience'. Of course, some may call the idea insane.

But then I started to almost panic about the idea being 'caught' in a potential endless sequence of ultimately

lonely and meaningless illusions.

I don't know if I am suffering from 'solipsism syndrome', but being around many people in public places usually makes me feel better.

My question is, if solipsism is (still) considered a philosophical challenge/ problem?

---

Most supposed refutations of solipsism work on the principle that there is no plausible explanation of why solipsism should be true; but they don't work because if solipsism is true then all explanations are false — because they explain by describing unperceived causes, which do not exist if solipsism is true. In principle there a re two ways to refute a particular position, such as solipsism: prove that it is contrary to empirical fact, or prove that it leads to a contradiction. In the case of solipsism, the first could be achieved if you could prove that something exists outside of your present consciousness; the difficulty of doing this is shown by the difficulty of proving the existence of God. A psychological explanation of why no one really believes solipsism is that we (rationally) believe in things in proportion to their power to explain, and solipsism's power of explanation is nil.

Helier Robinson

back

(98) Wassili asked:

My question is: what is or what means philosophy of art or aesthetics in the analytic tradition? Or better, in analytic tradition is a philosophical system incomplete until you have a theory of art? I understand the relevance of thinking the art as an aspect of culture, and I like authors like berger for example, and problems settled by artist like Kandinsky or Picasso, but what I do not understand is if it is necessary a philosophy oh art in order to complete a philosophical system in contemporary philosophy. Is the problem of representation (as Plato thought)relevant nowadays? I think that people like Greenberg, Kandinsky, Picasso and Duchamp showed how give up the art-reality problem. And they aren't philosophers. What can philosophy say about art despite the relations art history, art politics (as benjamin in the continental thinking)? There is something proper to philosophical thinking about art?

---

This is a good question and I would like to attempt an answer, even though I am not sure whether I am sufficiently qualified to talk about the state of art at the present time.

The first thing that needs to be said is that in the analytic tradition, one gives up aspirations towards a philosophical 'system'. Who is to say when, if ever, one can talk of a philosophy attaining the status of a system — i.e. being complete, coherent, all-encompassing?

Without doubt art is a problem for philosophy. It is proper for the philosopher to consider the nature of art, theories of aesthetic value, the aim and purpose of artistic endeavour — and even the relation between 'beauty' and 'truth' which I take to be a meaningful question.

What seems to me most remarkable is that art has reached an unprecedented point in its history where it has finally become indistinguishable from philosophy. I have heard of schools of art which no longer accept drawing or painting as a valid means of expression, but only conceptual work. To make an artistic 'statement' in conceptual terms is to make a philosophical point about the nature of art.

One wonders where things can go from here. Perhaps the history of art has finally come to an end. When every 'artist' is a philosopher, all that remains for 'artistic expression' in the traditional sense is in the realm popular culture — popular music, films, computer games etc.

Geoffrey Klempner

back

(99) Maha asked:

Considering the causal nature of the historical development of philosophy, is philosophy inextricably limited to the scope of historical works and achievements in the development of philosophical knowledge?

Specifically, I would like to question the possibilities of inspiration or innovation in philosophy relatively independent of historical works, and only influenced directly by the cultural and social environment in which we live. Specifically, how is one to approach/achieve the publication, and acceptance, of a novel philosophical theory, without quoted reference to previous works (possibly modern) in the relevant fields of philosophy; and how could one avoid the difficulties in clear and un-'tainted'' interpretation potentially caused by readers already causally linked to previous works in similarly categorisable fields of discussion of the subject. (I am assuming, I think correctly, that there will be some works which are of a similar set of considerations, to which there is likely to be mental reference and comparison made, to the detriment of the reasoning used in interpretation of the subject presented).

Specifically, my theoretical question of concern in this context is: 

assuming: the possibility of 'intrinsic variability' in the historical grounding (causationwise, literature-exposurewise, and interpretatively) to each philosophical idea ever considered, recorded, or discussed, 

is there: a plausible expectation that one's philosophical background cannot be variably dependent upon the causal association of variability in the reasoning of which ideas to contemplate when developing a 'philosophical idea', and hence 'philosophical reasoning' is inherently variably caused?

This question seems to contain a cyclical problem of induction specifically the assumption that reasoning be based upon causally variable intellectual/social background influences. I think I am correct in deducing that if the causal influence of the reasoning process is assumed to include not only the historical background of the influences, but also some causal variability in the method of thought or development of the reasoning (dependent upon the immediate environment of the thinker/reasoner, e.g. By distraction or inspiration), then it is possible to deduce that the causal variability of historical, intellectual and social influences gains its variability in causality in the immediate variability of the environment in which any philosophical thinking has/is taken/taking place.(?)

Also, I notice that this question, if comprehensible, suggests that your contemplation of it will be variable, dependent upon your philosophical background, but also upon the considerations made at the time of reasoning by your philosophical influences (I.e. Causally linked to the variability of reasoning in the recorded presentation of historical (possibly modern) works), and hence your answer will be causally dependent upon causally variable influences, but how is one to avoid the causally variable influence of your reply upon any non-historically based attempt to answer such a question without being variably causally influenced?

I seem to have reached an impossibility of reading your answer without failing in the attempt to achieve an answer without being variably influenced by it. Perhaps my interpretation will reach a reliably causally invariable state if my reading of your response is critically compensated. 

But thankyou if you try.

---

You seem to be tying yourself in knots here. The way out is to recognize that there are no tight causal connections but there are loose causal and rational connections.

Anyone who writes philosophy at a given time is subject to influence from what that person has gathered from his or her study of philosophy — however partial or careless or prejudiced that might be — as well as influence from the current culture which partly reflects some influence — however partial, careless or prejudiced that might be — from the history of philosophy.

However, if you want to make a contribution to philosophy there is no excuse whatsoever for being ignorant of the history of philosophy. You don't have to know everything — arguably, there can be such a thing as knowing too much which can crush creativity — but you need to know enough to know, e.g., That you are not unknowingly repeating another philosopher's errors.

Western philosophy has shown a remarkable coherence over the centuries, as well as startling discontinuities. Whatever choice there may be comes down, ultimately, to whether you see yourself as working within a tradition, or whether your view is more iconoclastic. Even so, as a would-be iconoclast, it is necessary to be aware of the tradition you are reacting against.

Every work in philosophy is part of an ongoing 'conversation', in the widest sense which does not necessarily imply a particular philosophical 'tradition'. If you are not willing to accept this minimum requirement of relevance, then what you are saying is that you don't care whether or not anyone reads what you have written or forms views about it.

Geoffrey Klempner

back

(100) Alex asked:

Hello! I have a question for you concerning propositional logic, a subject I have been studying for the past month in an effort to better understand philosophy as a whole. To teach myself logic I shopped around and eventually settled on an oft recommended text, which you may have heard of Logic, by Paul Tomassi. I have been reading through the book and doing the exercises, but have hit a wall with conditional proofs. I find I have no intuitive feel for how they work. I cant ask you to provide answers for all the questions, but here is one that has been nagging me.

Consider the sequent P, P : Q Now, I understand there is a rule that states a contradiction can lead to any result, which makes the above obviously correct. But I don't really understand what it all means. I have a feeling this may be a sort of trick to help us with complex proofs, sort of like the way I (as in, the square root of minus one) and other imaginary numbers are used in mathematics. But it shakes my understanding of just how the premises relate to one another. It is not helped by the fact that I can apply no real world argument to clarify things. If I take P as 2 + 2 = 4 and Q as it is raining, then we have

It is the case that 2 +2 = 4

It is not the case that 2 + 2 =4

Therefore

It is raining.

This argument strikes me as simply meaningless. How can a contradiction lead to an unrelated proposition in the same way that P v Q and P together lead to Q? And what does it mean to state both P and P together? Can we apply a semantic argument to this sequent? How?

---

You are quite right to be confused and frustrated with what are called the paradoxes of material implication: a false proposition implies any proposition and a true proposition is implied by any proposition. We simply do not think in such ways. These paradoxes come about because of the truth table for the conditional. If you check this truth table, and those for the other sentential connectives, you'll find that this truth table is the only one possible for implication — so modern logicians go along with it because the truth table approach to logic is so convenient. You are supposed to just avoid 'proving' anything with a false premise. My own view is that a paradox is a damning refutation of whatever leads to it, so that truth table logic should be abandoned. But I'm in a very small minority in that view.

Helier Robinson

back

(101) Maritza asked:

According to philosophy god does not exist since it is not proven with actual facts and documents. Well if this theory that god made us is false then there is the other theory that we evolved from monkeys if this is true then were did the monkeys come from since this question has not been answered. I ask you this question: where did all living things come from when this world was created according to the big bang theory?

---

It is not true that according to philosophy God does not exist. There are no agreed philosophical truths. Philosophers still argue about everything. Some philosophers believe in God, some do not. Some philosophers believe that you can prove that God exists, most philosophers think you can't prove that God exists. However even if you can't prove that God exists you might still believe in God.

Evolution is a scientific theory, it has nothing to do with philosophy. It is a scientific theory that is supported by an enormous amount of evidence. The theory of evolution does not contradict the idea that the world and everything in it were created by God but it does contradict the ideas of some fundamentalist Christians who want to interpret the Bible literally.

Most of the major Christian churches see no contradiction between evolution and belief that the Bible is the inspired word of God. The largest Christian church, the Catholic church, has even said officially that Catholics are free to believe in evolution.

The fact that you can ask a really silly question like 'Where did the monkeys come from?' Shows that you know nothing about the theory of evolution. Your first task should be to find a book that explains the theory in simple terms. You cannot criticise something you have not even bothered to study and expect people to take you seriously. So do some reading. When you understand the theory better and if you still have some questions ask them here.

Shaun Williamson

back

(102) lindsey asked:

Do you study the world.

---

Yes all the time. I am particularly surprised and fascinated by the three-dimensionality of the world.

Shaun Williamson

back

(103) Alex asked:

Hello! I have a question for you concerning propositional logic, a subject I have been studying for the past month in an effort to better understand philosophy as a whole. To teach myself logic I shopped around and eventually settled on an oft recommended text, which you may have heard of Logic, by Paul Tomassi. I have been reading through the book and doing the exercises, but have hit a wall with conditional proofs. I find I have no intuitive feel for how they work. I cant ask you to provide answers for all the questions, but here is one that has been nagging me.

Consider the sequent P, P : Q Now, I understand
there is a rule that states a contradiction can lead to any result, which makes the above obviously correct. But I don't really understand what it all means. I have a feeling this may be a sort of trick to help us with complex proofs, sort of like the way I (as in, the square root of minus one) and other imaginary numbers are used in mathematics. But it shakes my understanding of just how the premises relate to one another. It is not helped by the fact that I can apply no real world argument to clarify things. If I take P as 2 + 2 = 4 and Q as it is raining, then we have

It is the case that 2 +2 = 4

It is not the case that 2 + 2 =4

Therefore

It is raining.

This argument strikes me as simply meaningless. How can a contradiction lead to an unrelated proposition in the same way that P v Q and P together lead to Q? And what does it mean to state both P and P together? Can we apply a semantic argument to this sequent? How?

---

I think this shows how difficult it is to study logic on your own but at least you have made the correct start by concentrating on propositional logic.

The first thing you need to be clear about is that logic deals with logically valid arguments not necessarily relevant or sensible arguments. Is the idea of a logically valid argument important? It is the reason that we are able to build computers so it is extremely important.

The definition of a logically valid argument is that it is one where if the premises are true then it is impossible for the conclusion to be false. This is also the same as the truth conditions for the if...Then logical connective.

Now let's look at the general argument if 'p then q'. Is it valid, well we don't know. However lets look at the argument 'if p AND if not p then q'. Is this a valid argument. Well what we can say is that it is not possible for p and not p to both be true so our argument will never be valid even if q is true.

However if we admit that p and not p are both true then our argument if p then q AND if not p then q will be valid for any q. So we can derive any conclusion from a contradiction and this destroys the idea of a valid argument since we now have the means to make all arguments valid. Hope this helps but if not ask some further questions.

Shaun Williamson

back

(104) Theresa asked:

This is a question of Aristotle's laws of non contradiction.

That I have just discovered.Aristotle 'one cannot say something that it is and that it is not in the same respect and at the same time.

'Its plain that the same thing won't be willing at the same time to do or suffer opposites with respect to the same part and relation to the same thing.'

'Lets look at he statement 'Never say never.'

Meditating on this statement for quite some time now and relation to the to law.

I have discovered many intriguing points in it.

First lets get this thought out of the way.'What is the mathematical sequence of the phrase itself by the code of what lies in the subjective thought of sequence or contradiction within itself.Even though its not an object.And the truth is not in the objective to solve te riddle.One must look deeper

And even though it is separated by a word of 2 different points. It within itself expresses same part and will to the same respect. The person contradicts himself by saying never say never... But still does not fall under the coe of the law of non contradiction because the self and statement are 2 different parts. But if I say something that contradicts itself with the thought of 2 different directions of relation.. It still doesn't fall under the law of non contradiction. But this I see as the same part..Now we have to look at. Is it a logical statement that has value? Is it true. Is it logical to suggest to someone.. I think so... So does this break the laws of Aristotle's law of non contradiction... And is he only objective minded and really not of the higher intellect. Aristotle is intelligent and has an analytical mind. But how limited is it? So does this observation of mine fall under(asking from another point of view )... Proving the laws of non contradiction as incomplete?

---

Well the law of non contradiction is one that applies to statements about the world such as 'It is raining' or 'It is not raining'. 'Never say never' isn't exactly a statement so I think the law of non contradiction is still safe.

However what you have noticed in a round about sort of way is the logical peculiarity of self referential statements. An ordinary self referential statement would be something like 'This sentence has six words', which is in fact false. A more paradoxical self referential statement is something like 'John said that he never uttered the word never'. There is an ancient Greek paradox about self referential statements that became extremely important to the development of logic in the 20th century, it goes like this. A Greek who is thinking of travelling to Crete meets a Cretan and asks him what the people on his island are like. He replies 'All Cretans always tell lies'. The problem is to decide if what he has told you is true because if it's true then it's false so it must be true therefore it's false and so on.

The problem suggested by paradoxes like this gave one of the greatest logicians, Kurt Godel, the idea for his proof that we can never prove that mathematics is both consistent and complete.

Shaun Williamson

back

(105) Aquilinus asked:

Why is it that there is more evil deeds in the world than good deeds and yet overwhelming number of people don't like evil?

---

A pessimist might say that there are more evil deeds than good deeds. An optimist might think the opposite. I don't see how we can possibly work out which one is true. However consider the following. The group in society who most fear being attacked and robbed in the street, are old people. Yet they are the group least likely to be attacked and robbed. What we imagine is true and what is actually true are often quite different.

Shaun Williamson

back

(106) Kay asked:

I believe every human being on this planet has lied sometime in life... So are we all not liars?

---

It is truistic that anyone who has ever told a lie is a liar, just as anyone who has ever stolen anything is a thief.

However, there is a difference between being a 'liar' or a 'thief' in this sense and being a habitual liar or a habitual thief. Thus, you can be a 'liar' in the sense of having told a lie at some time, without being a habitual liar.

We also have to allow for the possibility of a reformed character. Someone who was once a habitual liar or a habitual thief can reform, and give up lying and thieving.

The underlying point of your question is well taken: when we make a fuss about the wrongs committed by others, we ought to remember our own failings.

Geoffrey Klempner

back

(107) Phil asked:

It seems clear to me that there are objective moral values, that is, that certain moral values exist regardless of whether anyone agrees with them or not.

If we take this as granted, is there a coherent theory within a metaphysical naturalist framework as to why (a) they exist and (b) why people should be obliged to live according to such values?

(I know some people argue that there are no objective moral values, but I'm trying to work out the best argument if you think (a) they exist and (b) metaphysical naturalism is true). Your help would be appreciated!!

---

I think that it is possible to argue for an objective basis for moral conduct, within a naturalist framework. This is not quite the same as a belief in objective moral values, either in the Platonic sense of objectively existing moral Forms like Justice, or in the Kantian sense of an objective criterion for moral conduct, the so-called Categorical Imperative.

For this argument we need to assume, for the sake of reductio ad absurdum, an agent who does not recognize that there is any reason for taking other persons into consideration when they act: an 'amoralist'. If there is no objective basis for moral conduct, then there is no contradiction or incoherence in the idea of a person who consciously chooses to live the life of an amoralist.

Here is a brief summary of my argument, adapted from my paper 'In Pursuit of the Amoralist' at http://klempner.freeshell.org/articles/shap2.html

step 1: I inhabit a world. The world is not my dream, not a story I make up as I go along. I am in the world.

Step 2: My factual judgements can be true or false and not merely more or less useful from my point of view. Even if I refuse to recognize when 'the world proves me wrong', I can be wrong, all the same.

Step 3: It follows from the fact that I conceive myself as being in the world — from the fact that I am not a solipsist — that other people are more than my measuring instruments.

Step 4: Truth, or the truth as such is what is common: a truth that is the same for me as it is for other persons who share that world with me: the truth as such.

Step 5: It follows that what the statement, 'Other persons are not my measuring instruments' is meant to describe is a condition for the possibility of truth.

Step 6: As that which makes truth possible, what is meant by the statement, 'Other persons are not my measuring instruments' cannot, literally, be said. Nor can it be judged or believed.

Step 7: If 'Other persons are not my measuring instruments' cannot be said, then it must be shown in some other way than by saying.

Step 8: The only way to show that other persons are not my measuring instruments is by my actions.

Step 9: The only action which will show this is a moral action, an action motivated by my perception of the legitimate claims of the other person, irrespective of my own desires or interests regarding that person.

- I would regard this argument as less than logically conclusive. What it does is raise the price of being an amoralist to a point which is too high for all but the most determined sceptic. When you look at all you have to give up in order to be a consistent amoralist, then the rationally preferable course is to recognize that there is, indeed, an objective basis for moral conduct.

Geoffrey Klempner

back

(108) Adam asked:

Would you agree that reality is like a place holder ? Reality
is the state of affairs that holds all other state of affairs ?

Do you see a point in talking this way ?

---

I don't really see any point in talking in this was but it does have some relation to logic. If we take all the possible propositions that can be formed then some of them will be true and some will be false. Each true one will correspond to a state of affairs that exists in the world and this is what reality is rather than all possible state of affairs. All possible states of affairs are defined by the total logical propositional space.I.e. Both true (real) and false (not real).

Shaun Williamson

back

(109) Wassili asked:

My question is: what is or what means philosophy of art or aesthetics in the analytic tradition? Or better, in analytic tradition is a philosophical system incomplete until you have a theory of art? I understand the relevance of thinking the art as an aspect of culture, and I like authors like berger for example, and problems settled by artist like Kandinsky or Picasso, but what I do not understand is if it is necessary a philosophy oh art in order to complete a philosophical system in contemporary philosophy. Is the problem of representation (as Plato thought)relevant nowadays? I think that people like Greenberg, Kandinsky, Picasso and Duchamp showed how give up the art-reality problem. And they aren't philosophers. What can philosophy say about art despite the relations art history, art politics (as Benjamin in the continental thinking)? There is something proper to philosophical thinking about art?

---

Unfortunately the analytic tradition is too diverse to have any common approach to aesthetics. However the non-analytic tradition also has nothing in common with regard to these questions. Kantians for example think that philosophy can provide a rational basis for value judgements while the various sort of existentialists take it for granted that philosophy cannot provide such a rational basis for value judgements and this would apply equally to aesthetics and ethics.

In the analytic tradition there are the positivists such the early A.J. Ayer and the Vienna Circle who were inclined to view value judgements as expressions of feeling. Wittgenstein in his early work regarded value judgements as being beyond the scope of philosophy. In his later work he held that philosophy cannot provide a rational basis for value judgements because value judgements do not need such justification. Berger was a Marxist so more properly belongs to the non-analytic tradition.

The only thing that the different analytic philosophers have in common is an unwillingness to indulge in speculative theorising, preferring instead to concentrate on the unsolved problems of philosophy.

Shaun Williamson

back

(110) Alex asked:

Hello! I have a question for you concerning propositional logic, a subject I have been studying for the past month in an effort to better understand philosophy as a whole. To teach myself logic I shopped around and eventually settled on an oft recommended text, which you may have heard of Logic, by Paul Tomassi. I have been reading through the book and doing the exercises, but have hit a wall with conditional proofs. I find I have no intuitive feel for how they work. I cant ask you to provide answers for all the questions, but here is one that has been nagging me.

Consider the sequent P, P : Q Now, I understand there is a rule that states a contradiction can lead to any result, which makes the above obviously correct. But I don't really understand what it all means. I have a feeling this may be a sort of trick to help us with complex proofs, sort of like the way I (as in, the square root of minus one) and other imaginary numbers are used in mathematics. But it shakes my understanding of just how the premises relate to one another. It is not helped by the fact that I can apply no real world argument to clarify things. If I take P as 2 + 2 = 4 and Q as it is raining, then we have:

It is the case that 2 +2 = 4

It is not the case that 2 + 2 =4

Therefore

It is raining.

This argument strikes me as simply meaningless. How can a contradiction lead to an unrelated proposition in the same way that P v Q and P together lead to Q? And what does it mean to state both P and P together? Can we apply a semantic argument to this sequent? How?

---

Consider the following:

2+2 = 4
Therefore (2+2=4) or (I am a monkey's uncle.)

Anything can go in the second bracket, because only one bracket contents has to be true for the statement to be true — that's what 'or' means, and we already know that the first is.

It is not the case that (2+2=4).

Now we contradict the first statement, and deny the first bracket in the conditional.

Therefore (I am a monkey's uncle.) QED.

Normally, the two statements in a conditional are related, so it seems peculiar; e.g. 'This egg will break when I drop it, or this egg is hard-boiled.' If I drop the egg onto a pillow, the statement might be false (if the egg is not hard-boiled). In that case I might say 'This egg will break, or I am standing over a pillow.' The 2 statements in this conditional are not related, except through circumstance, yet we can make sense of it.

Shaun Williamson

back

(111) Maha asked:

Considering the causal nature of the historical development of philosophy, is philosophy inextricably limited to the scope of historical works and achievements in the development of philosophical knowledge?

Specifically, I would like to question the possibilities of inspiration or innovation in philosophy relatively independent of historical works, and only influenced directly by the cultural and social environment in which we live. Specifically, how is one to approach/achieve the publication, and acceptance, of a novel philosophical theory, without quoted reference to previous works (possibly modern) in the relevant fields of philosophy; and how could one avoid the difficulties in clear and un-'tainted'' interpretation potentially caused by readers already causally linked to previous works in similarly categorisable fields of discussion of the subject. (I am assuming, I think correctly, that there will be some works which are of a similar set of considerations, to which there is likely to be mental reference and comparison made, to the detriment of the reasoning used in interpretation of the subject presented).

Specifically, my theoretical question of concern in this context is:

assuming: the possibility of 'intrinsic variability' in the historical grounding (causationwise, literature-exposurewise, and interpretatively) to each philosophical idea ever considered, recorded, or discussed,

is there: a plausible expectation that one's philosophical background cannot be variably dependent upon the causal association of variability in the reasoning of which ideas to contemplate when developing a 'philosophical idea', and hence 'philosophical reasoning' is inherently variably caused?

This question seems to contain a cyclical problem of induction
specifically the assumption that reasoning be based upon causally variable intellectual/social background influences.

I think I am correct in deducing that if the causal influence of the reasoning process is assumed to include not only the historical background of the influences, but also some causal variability in the method of thought or development of the reasoning (dependent upon the immediate environment of the thinker/reasoner, e.g. By distraction or inspiration), then it is possible to deduce that the causal variability of historical, intellectual and social influences gains its variability in causality in the immediate variability of the environment in which any philosophical thinking has/is taken/taking place.(?)

Also, I notice that this question, if comprehensible, suggests that your contemplation of it will be variable, dependent upon your philosophical background, but also upon the considerations made at the time of reasoning by your philosophical influences (I.e. Causally linked to the variability of reasoning in the recorded presentation of historical (possibly modern) works), and hence your answer will be causally dependent upon causally variable influences, but how is one to avoid the causally variable influence of your reply upon any non-historically based attempt to answer such a question without being variably causally influenced?

I seem to have reached an impossibility of reading your answer without failing in the attempt to achieve an answer without being variably influenced by it. Perhaps my interpretation will reach a reliably causally invariable state if my reading of your response is critically compensated.

But thankyou if you try.

---

There is a proof which is over 2000 years old that there is no greatest prime number. This proof tells us of a truth which can never be changed or contradicted and will remain true forever. We don't worry about the role of influence or causality when we contemplate this truth.

People will often try to persuade you that truth is relative but this is nonsense. Two plus two equals four is true and as a truth it is not relative to anything. In the same way philosophy is about truth so forget causality and influence. Forget about the body of philosophical knowledge, there is no body of philosophical knowledge and there never will be.

Just think about truth and falsity.

Shaun Williamson

back

(112) Kay asked:

I believe every human being on this planet has lied sometime

in life...So are we all not liars?

---

Probably everyone has lied at some time since children at certain ages do not know the difference between truth and lies. So if your definition of a liar is someone who has told at least ONE lie during their life then we are all liars. However everyone knows this and it really isn't very interesting. You might just as well argue that because everyone has had at least one pimple during their life that we all have bad skin.

There is a difference between people who lie for their own gain, people who lie to avoid causing pain to others and people who lie for fun. I suggest you think about the differences between liars.

Shaun Williamson

back

(113) India asked:

I would be interested to know why the book 'Catcher in the Rye' had such a profound influence on its readers...?

---

It was a book that exactly fitted the time in which it was written. The hero was a teenager in an era that was about to become dominated by teenagers. He was an existential hero looking for authenticity in an age where young people were attracted by existential philosophy. His inability to conform to what he saw as the hypocrisy of the adult society around him appealed to a young generation who wanted a social revolution which would make society more open to new ideas about relationships and marriage.

Shaun Williamson

back

(114) Justin asked:

When keeping in mind why and how we do things, do you think that we actually get to create /choose our lives or are we like a marble on a ramp, that will travel a certain speed (according to the angle and weight of the marble)and a certain distance according to the factors at hand.

---

A marble on a ramp knows nothing about marbles and ramps. We can know the consequences of our actions. We can even know the laws of physics to some extent. We have wiped out smallpox, a marble has never done anything like that. A marble has never painted a picture, written a poem or a novel.

It would seem strange to ever compare the course of our lives to something so dissimilar to ourselves.

Shaun Williamson

back

(115) Phil asked:

It seems clear to me that there are objective moral values, that is, that certain moral values exist regardless of whether anyone agrees with them or not.

If we take this as granted, is there a coherent theory within a metaphysical naturalist framework as to why (a) they exist and (b) why people should be obliged to live according to such values?

(I know some people argue that there are no objective moral values, but I'm trying to work out the best argument if you think (a) they exist and (b) metaphysical naturalism is true). Your help would be appreciated!!

---

The answer is — Yes there is. It is Evolutionary Ethics. You can find numerous entries using Google. You might also find this site of interest:

http://www3.sympatico.ca/saburns/pg0410.htm

Stuart Burns

back

(116) Derrick asked:

Why is philosophy so confusing?

---

Philosophy is not confusing!

What some people find unfamiliar, and hence perhaps a little strange, is that Philosophy backs away from specific detail to ask more generalized questions. You might be familiar and comfortable with such questions as 'Is abortion right or wrong?'. But you might find a little unsettling such philosophical questions as 'On what basis, according to what principles or guidelines, do you reach that evaluation?' Most people depend for their moral evaluations on the dictates of others — parents and role models; religious leaders; a 'sacred' text, and do forth. Hence, most people make their moral choices on the basis of the dictates of others. Few people are comfortable examining the fundamental underlying foundations of their moral choices.

If you find Philosophy confusing, perhaps it is because you find that it is challenging some of your own personal 'sacred cows'.

Stuart Burns

back

(117) Claire asked:

Isn't Ayn Rand's notion of free will a contradiction? She believes that man has free will and she rejects determinism. But I also read that she believes that free will is not contradictory to the law of causality. Doesn't determinism rest on cause and effect as well? So what's the difference or how can Objectivism meet this?

---

Ayn Rand is what is generally known as a 'Compatibilist' — some one who maintains that there is no contradiction between Free Will (properly understood), and Determinism as the level of physical causality. I am no objectivist, so I cannot comment in detail on Rand's compatibilist reasoning. But if you would like to see a compatibilist argument, check out this page: http://www3.sympatico.ca/saburns/pg0407.htm

Stuart Burns

back

(118) Danilo asked:

Hi, I would appreciate an answer from anybody but I will reference somethings that Stuart Burns said in 38/15:

1) Is there intraspecies altruism between non-humans? Is a being 'X' of species 'A' capable of an altruistic act towards a being 'Y' of the same species?

2)Is there interspecies altruism between non-humans? Is a being 'X' of species 'A' capable of an altruistic act towards a being 'Y' of species 'B'?

Assuming the answer to 38/15 to be true and the answer to 1) is 'yes', I believe the answer to 38/15 is still valid because the act can benefit the species A as a whole.

If the answer to 2) is 'yes' and the act does not result in any benefit to any member of species A, is it possible to say that non-humans do not always 'fulfill their purpose in life'?

---=

In order to answer your question, we first require a definition of altruism. Specifically, we need to characterize what would constitute 'an altruistic act'. Now according to my favourite dictionary, 'altruism' is 'an attitude or way of behaving marked by unselfish concern for the welfare of others.' So I think it would be fair to characterize 'an altruistic act' of X towards Y as a act that is performed by X with the specific intention of benefiting Y at some (possibly negligible) cost to X. Now considerable care must be taken here, of course, because we are trying to apply the notion of 'intention' to non-human species. On the surface, it would appear that non-human species are incapable of an altruistic act because a non-human species is incapable of the necessary 'specific intention of benefiting Y'. But we can relax the requirements a little, and suggest that the necessary non-human intention can be taken to exist if the behavior being elicited would have involved intention if acts had been performed by a human.

(1) Given that two part definition, is a being 'X' of species 'A' capable of an altruistic act towards a being 'Y' of the same species? I would suggest that the answer is a qualified Yes. I qualify the 'Yes', because evolution happens at the genetic level, and not at the individual being level. So a mother cat dying to save her kittens would be an altruistic act that the individual level, but non-altruistic at the genetic level. What you normally do not get in non-human species, however, is a mother cat dying to save genetically unrelated kittens. (You can, of course, manipulate the conditions so as to confuse the evolutionary imperatives — if you swap a mother cat's litter immediately after birth with a genetically unrelated litter, then it is possible that the mother cat will be fooled into treating her surrogate litter as her own, and might then die to save the genetically unrelated kittens she believes are hers.)

(2) Again, given our definition of 'an altruistic act', is a being 'X' of species 'A' capable of an altruistic act towards a being 'Y' of species 'B'? I would suggest that the answer is a qualified No. To any being X, the members of other species would constitute the local environment. Evolutionary pressures would seem to filter out any tendency of any being to intentionally sacrifice its own welfare on behalf of some member of another species. Which is not to argue against the obvious, of course, that many species can and do cooperate together for mutual benefit — but that is not altruism. The only qualification that I would place on my 'no' is again due to the fact that evolution happens at the genetic level and not the individual level. The Lancet liver fluke (Dicrocoelium dendriticum) is a parasite that infests a species of ant. Infected ants climb to the top of grass stalks in order to make it easier for the parasite fluke to transition to its next host — grazing ungulates like cattle or sheep. Is the ant's behavior an altruistic act on behalf of the parasite fluke? On the one hand, clearly not because there is no intention involved. The ant's behavior is dictated by the fluke. But on the other hand, our relaxation of the requirements for 'intention' might allow us to classify the ant's behavior as suitably intentional, and hence altruistic.

(3) Is it possible to say that non-humans do not always 'fulfill their purpose in life'? I think the answer is an obvious and unqualified 'Yes'. But again, it is a matter of the level of discussion. Individuals, like that parasite infested ant, might not always fulfill their purpose in life. Evolution takes place when there are insufficient resources in the environment to permit every individual from generating as many descendents as mathematically possible in principle. Inevitably, because of differences in genetic endowment, geographic enrichment, and sheer accident, some individuals will be more successful than others. 'Shit Happens!' But then evolution takes place at the genetic level. And the future will be populated by the descendents of individuals who do successfully manage to fulfill their purpose in life. Now, is it possible to say that non-humans do not always intend to 'fulfill their purpose in life'? I think that here the answer must be the same sort of qualified 'No' as was given for answer (2) above. At the level of the individual being, it may be possible to find or manufacture examples that we might be happy to call altruistic. But at the genetic level, I don't think we can stretch the interpretation of 'intent' sufficiently to allow that genetics can 'intend' to cause the individual to act altruistically. And at the genetic level, evolutionary pressures would quickly ensure that such self-sacrificing behavior was out competed by those who were not so inclined.

Stuart Burns

back

(119) Alex asked:

Hello, I have reached a conclusion that is quite dangerous to my health and could lead to a lot of trouble. I need to ask someone and see if they come to the same conclusion. My question is: are you the same person you were 1 year ago or even 5 minutes ago?

I figured that the self changes over time, regarding both personality and physical appearance. As you gain knowledge and change your opinion, your personality changes and you seem to be totally different then you were before. Your physical appearance also changes over time, the cells in your body completely replace themselves in about 7 years (i think). Although your memory really doesn't change over time, only how you perceive this memory does, and how you perceive the world around you. To further define my question: because we are constantly changing and are becoming a new person (except for our memory which ties our life together and gives us the illusion that we are the same person) should I be living completely in the present and totally disregarding the past and future?

You can see what problems this creates. Thanks you.

---

I do not see how your conclusion (I should be living completely in the present and totally disregarding the past and future) follows from your premise (the self changes over time, regarding both personality and physical appearance).

Sure we are all always changing — in personality, memory, physical appearance, physical makeup, and so forth. But so what? How does that fact suggest that the past and the future do not matter?

You are, over time and despite al those changes, two interconnected and persistent patterns — a pattern of connected memories, and a pattern of DNA. Despite all those changes, those two patterns remain consistent, persistent, and interconnected. That is what you really are. And to maintain, sustain, and ensure the continued existence of those patterns, you need to pay careful attention to both the past and the future, as well as the present.

Stuart Burns

back

(120) Tim asked:

Is is possible to have a finite amount of matter in an infinite universe?

---

Sure.

If a finite amount of matter is spread uniformly across an infinite universe, then any finite region of that universe would contain only an infinitesimal amount of matter. And if we assume, as would seem reasonable given quantum mechanics, that there is a minimum amount of matter that could exist — a quantum of matter; then there would be a finite number of infinitely large regions of that universe each containing a single quantum of matter, along with an infinite number of infinitely large regions that contain no matter at all. Each single quantum of matter would be infinitely distant from any other quantum. (The mathematics of infinity is fascinatingly strange.)

On the other hand, if we allow the finite amount of matter to 'clump' together, then we could have a finite number of areas in the universe that look similar to our own visible horizon. Each clump, of course, would be infinitely distant from any other clump.

Stuart Burns